Top Banner
INSIGHTSIAS SIMPLYFYING IAS EXAM PREPARATION INSTA Tests 65 to 68 (GS) www.insightsonindia.com prelims.insightsonindia.com | mains.insightsonindia.com Telegram: insightsIAStips | FB: insightsonindia | TW: vinaygb | YT: INSIGHTS IAS BENGALURU | DELHI | HYDERABAD INSTA 75 Days REVISION PLAN UPSC Prelims 2020 Copyright © by Insights IAS All rights are reserved. No part of this document may be reproduced, stored in a retrieval system or transmitted in any form or by any means, electronic, mechanical, photocopying, recording or otherwise, without prior permission of Insights IAS. KEY & EXPLANATIONS
105

SIMPLYFYING IAS EXAM PREPARATION - INSIGHTSIAS · Insta 75 Days Revision Plan for UPSC Civil Services Prelims – 2020 This document is the compilation of 100 questions that are part

Jul 05, 2020

Download

Documents

dariahiddleston
Welcome message from author
This document is posted to help you gain knowledge. Please leave a comment to let me know what you think about it! Share it to your friends and learn new things together.
Transcript
Page 1: SIMPLYFYING IAS EXAM PREPARATION - INSIGHTSIAS · Insta 75 Days Revision Plan for UPSC Civil Services Prelims – 2020 This document is the compilation of 100 questions that are part

INSIGHTSIAS SIMPLYFYING IAS EXAM PREPARATION

INSTA Tests

65 to 68 (GS)

www.insightsonindia.com

prelims.insightsonindia.com | mains.insightsonindia.com

Telegram: insightsIAStips | FB: insightsonindia | TW: vinaygb | YT: INSIGHTS IAS

BENGALURU | DELHI | HYDERABAD

INSTA 75 Days REVISION PLAN UPSC Prelims 2020

Copyright © by Insights IAS All rights are reserved. No part of this document may be reproduced, stored in a retrieval system or transmitted in any form or by any means, electronic, mechanical, photocopying, recording or otherwise, without prior permission of Insights IAS.

KEY & EXPLANATIONS

Page 2: SIMPLYFYING IAS EXAM PREPARATION - INSIGHTSIAS · Insta 75 Days Revision Plan for UPSC Civil Services Prelims – 2020 This document is the compilation of 100 questions that are part

Insta 75 Days Revision Plan for UPSC Civil Services

Prelims – 2020

This document is the compilation of 100 questions that are part of InsightsIAS

famous INSTA REVISION initiative for UPSC civil services Preliminary examination

– 2020 (which has become most anticipated annual affair by lakhs of IAS aspirants

across the country). These questions are carefully framed so as to give aspirants tough

challenge to test their knowledge and at the same time improve skills such as

intelligent guessing, elimination, reasoning, deduction etc – which are much needed

to sail through tough Civil Services Preliminary Examination conducted by UPSC.

These questions are based on this INSTA Revision Plan which is posted on our

website (www.insightsonindia.com). Every year thousands of candidates follow our

revision timetable – which is made for SERIOUS aspirants who would like to intensively

revise everything that’s important before the exam.

Those who would like to take up more tests for even better preparation, can

enroll to Insights IAS Prelims Mock Test Series – 2020

(https://prelims.insightsonindia.com). Every year toppers solve our tests and sail

through UPSC civil services exam. Your support through purchase of our tests will help

us provide FREE content on our website seamlessly.

Wish you all the best!

Team InsightsIAS

Page 3: SIMPLYFYING IAS EXAM PREPARATION - INSIGHTSIAS · Insta 75 Days Revision Plan for UPSC Civil Services Prelims – 2020 This document is the compilation of 100 questions that are part

INSTA 75 Days REVISION PLAN for Prelims 2020 - InstaTests

www.insightsonindia.com 1 Insights IAS

DAY – 65 (InstaTest-65)

1. Consider the following statements regarding International Treaty on Plant Genetic

Resources for Food and Agriculture

1. It is also known as the International Seed Treaty.

2. The Treaty helps maximize the use and breeding of all crops and promotes

development and maintenance of diverse farming systems.

3. This treaty recognizes and protects the farmer rights.

Which of the statements given above is/are correct?

(a) 1 and 2 only

(b) 1 and 2 only

(c) 3 only

(d) 1, 2 and 3

Solution: D

• International Treaty on Plant Genetic Resources for Food and Agriculture is popularly

known as the International Seed Treaty, is a comprehensive international agreement

in harmony with the Convention on Biological Diversity, which aims at guaranteeing

food security through the conservation, exchange and sustainable use of the world’s

plant genetic resources for food and agriculture (PGRFA), as well as the fair and

equitable benefit sharing arising from its use.

It also recognizes farmers’ rights, subject to national laws to:

• the protection of traditional knowledge relevant to plant genetic resources for food

and agriculture;

• the right to equitably participate in sharing benefits

• the right to participate in making decisions etc.

The Treaty helps maximize the use and breeding of all crops and promotes development

and maintenance of diverse farming systems.

2. Consider the following statements regarding Repco Bank

1. Repco Bank is a multi-state cooperative society established for rehabilitation of

repatriates from Myanmar and Sri Lanka.

2. It is controlled by Ministry of Finance.

Page 4: SIMPLYFYING IAS EXAM PREPARATION - INSIGHTSIAS · Insta 75 Days Revision Plan for UPSC Civil Services Prelims – 2020 This document is the compilation of 100 questions that are part

INSTA 75 Days REVISION PLAN for Prelims 2020 - InstaTests

www.insightsonindia.com 2 Insights IAS

3. It is operated only in the South Indian states of Andhra Pradesh, Karnataka, Kerala

and Tamil Nadu.

Which of the statements given above is/are correct?

(a) 1 only

(b) 2 and 3 only

(c) 1 and 3 only

(d) 1, 2 and 3

Solution: C

Repco Bank

• Union Home Minister received a dividend cheque of Rs 15.26 crore from the

representatives of the Repco Bank, a multi-state cooperative finance and

development bank controlled by the Home Ministry.

• The Repco Bank is a multi-state cooperative society established in 1969 by the central

government for rehabilitation of repatriates from Myanmar and Sri Lanka.

• It is operated only in the South Indian states of Andhra Pradesh, Karnataka, Kerala and

Tamil Nadu.

• As on March 31, 2019 the government of India held 49.15 per cent of the share capital,

four southern state governments held 6.24 per cent and the remaining 45 per cent

was held by individual repatriates.

• Repatriates Cooperative Finance and Development Bank Ltd., (REPCO BANK) was

registered on 19.11.1969 as a Cooperative Society under the relevant provisions of

Madras Cooperative Societies Act, 1961 with Jurisdiction over the State of Tamil

Nadu, Andhra Pradesh, Karnataka, Kerala and Union Territory of Puducherry for the

purpose of promoting the rehabilitation activities for repatriates from neighboring

countries mainly from Sri Lanka and Burma. Though originally registered under the

Madras Cooperative Societies Act, 1961, the Bank is deemed to be registered under

the Multi State Cooperative Societies Act, 2002.

• The share capital of the Bank is controlled by Government of India, Governments of

4 Southern States namely Tamil Nadu, Kerala, Karnataka, Andhra Pradesh and

Repatriates.

Page 5: SIMPLYFYING IAS EXAM PREPARATION - INSIGHTSIAS · Insta 75 Days Revision Plan for UPSC Civil Services Prelims – 2020 This document is the compilation of 100 questions that are part

INSTA 75 Days REVISION PLAN for Prelims 2020 - InstaTests

www.insightsonindia.com 3 Insights IAS

3. Consider the following statements regarding Western Dedicated Freight Corridor (DFC)

1. It begins at Dadri in Uttar Pradesh and till the Jawaharlal Nehru Port Trust, near

Mumbai.

2. The Dedicated Freight Corridor Corporation of India (DFCCIL) has been established

to carry out the project.

3. The project will be funded by loan from World Bank.

Which of the statements given above is/are correct?

(a) 1 and 2 only

(b) 2 and 3 only

(c) 1 only

(d) 1, 2 and 3

Solution: A

Western Dedicated Freight Corridor (DFC)

• It is a broad-gauge corridor.

• The 1,504-km western freight corridor begins at Dadri in Uttar Pradesh and stretches

till the country’s largest container port — Jawaharlal Nehru Port Trust, near Mumbai.

• In October 2006, a dedicated body, the Dedicated Freight Corridor Corporation of

India (DFCCIL) has been established to carry out the project.

• The project will be funded by a soft loan of $4bn provided by Japan International

Cooperation Agency under special terms for economic partnership (STEP).

Important places in this route:

• It passes through Vadodara, Ahmadabad, Palanpur, Phulera and Rewari.

• Passing through U.P., Haryana, Rajasthan, Gujarat and Maharashtra.

The project will eventually be linked to the Eastern DFC to form four hubs known as India’s

Golden Quadrilateral including Delhi, Mumbai, Chennai and Kolkata.

About Dedicated Freight Corridor Corporation of India (DFCCIL): The DFCCIL is a corporation

run by the Ministry of Railways (India) to undertake planning & development, mobilization

of financial resources and construction, maintenance and operation of the Dedicated Freight

Corridors. DFCC has been registered as a company under the Companies Act 1956 on 30

October 2006.

Page 6: SIMPLYFYING IAS EXAM PREPARATION - INSIGHTSIAS · Insta 75 Days Revision Plan for UPSC Civil Services Prelims – 2020 This document is the compilation of 100 questions that are part

INSTA 75 Days REVISION PLAN for Prelims 2020 - InstaTests

www.insightsonindia.com 4 Insights IAS

4. Consider the following statements regarding Jal Jeevan Mission

1. It aims to create local infrastructure for rainwater harvesting, groundwater

recharge and management of household waste water for reuse in agriculture.

2. The chief objective of the mission is to provide piped water supply to all rural and

urban households by 2024.

3. India has 20% of freshwater resources of the entire world.

Which of the statements given above is/are correct?

(a) 1 and 3 only

(b) 2 only

(c) 1 and 2 only

(d) 1, 2 and 3

Solution: C

• Jal Jeevan Mission was announced in August 2019.

• The chief objective of the Mission is to provide piped water supply (Har Ghar Jal) to

all rural and urban households by 2024.It also aims to create local infrastructure for

rainwater harvesting, groundwater recharge and management of household waste

water for reuse in agriculture

• India has 16% of the world population, but only 4% of freshwater resources. Depleting

groundwater level, overexploitation and deteriorating water quality, climate change,

etc. are major challenges to provide potable drinking water

• Jal Jeevan Mission will focus on integrated demand and supply management of water

at the local level.

5. Consider the following statements regarding STRIPS – Separate Trading of Registered

Interest and Principal of Securities

1. STRIPS are the securities created by way of separating the cash flows associated

with a regular G-Sec and the final principal payment to be received from the

issuer, into separate securities.

2. They are essentially Zero-Coupon Bonds (ZCBs).

3. They are created out of existing securities only and like other securities, they are

issued through auctions.

Which of the statements given above is/are correct?

(a) 1 and 2 only

(b) 2 and 3 only

Page 7: SIMPLYFYING IAS EXAM PREPARATION - INSIGHTSIAS · Insta 75 Days Revision Plan for UPSC Civil Services Prelims – 2020 This document is the compilation of 100 questions that are part

INSTA 75 Days REVISION PLAN for Prelims 2020 - InstaTests

www.insightsonindia.com 5 Insights IAS

(c) 1 only

(d) 1, 2 and 3

Solution: A

STRIPS – Separate Trading of Registered Interest and Principal of Securities. – STRIPS are the

securities created by way of separating the cash flows associated with a regular G-Sec i.e.

each semi-annual coupon payment and the final principal payment to be received from the

issuer, into separate securities. They are essentially Zero-Coupon Bonds (ZCBs). However,

they are created out of existing securities only and unlike other securities, are not issued

through auctions. Stripped securities represent future cash flows (periodic interest and

principal repayment) of an underlying coupon bearing bond. Being G-Secs, STRIPS are

eligible for SLR. All fixed coupon securities issued by Government of India, irrespective of the

year of maturity, are eligible for Stripping/Reconstitution, provided that the securities are

reckoned as eligible investment for the purpose of Statutory Liquidity Ratio (SLR) and the

securities are transferable. The detailed guidelines of stripping/reconstitution of government

securities is available in RBI notification IDMD.

For example, when ₹100 of the 8.60% GS 2028 is stripped, each cash flow of coupon (₹ 4.30

each half year) will become a coupon STRIP and the principal payment (₹100 at maturity) will

become a principal STRIP. These cash flows are traded separately as independent securities

in the secondary market. STRIPS in G-Secs ensure availability of sovereign zero coupon bonds,

which facilitate the development of a market determined zero coupon yield curve (ZCYC).

STRIPS also provide institutional investors with an additional instrument for their asset

liability management (ALM). Further, as STRIPS have zero reinvestment risk, being zero

coupon bonds, they can be attractive to retail/non-institutional investors. Market

participants, having an SGL account with RBI can place requests directly in e-kuber for

stripping/reconstitution of eligible securities (not special securities). Requests for

stripping/reconstitution by Gilt Account Holders (GAH) shall be placed with the respective

Custodian maintaining the CSGL account, who in turn, will place the requests on behalf of its

constituents in e-kuber.

6. Consider the following statements regarding Complaint Management System (CMS)

1. It is a software application to facilitate RBI’s grievance redressal process.

2. Customers can lodge complaints only against commercial banks.

3. The complaint would be directed to the appropriate office of the Ombudsman.

Which of the statements given above is/are correct?

(a) 1 and 2 only

Page 8: SIMPLYFYING IAS EXAM PREPARATION - INSIGHTSIAS · Insta 75 Days Revision Plan for UPSC Civil Services Prelims – 2020 This document is the compilation of 100 questions that are part

INSTA 75 Days REVISION PLAN for Prelims 2020 - InstaTests

www.insightsonindia.com 6 Insights IAS

(b) 2 and 3 only

(c) 1 and 3 only

(d) 1, 2 and 3

Solution: C

Complaint Management System (CMS)

Launched by the Reserve Bank of India (RBI), the Complaint Management System (CMS) is a

software application to facilitate RBI’s grievance redressal process.

• Aim: to improve customer experience in timely redressal of grievances.

How it works?

• Customers can lodge complaints against any regulated entity with public interface

such as commercial banks, urban cooperative banks, Non-Banking Financial

Companies (NBFCs). The complaint would be directed to the appropriate office of the

Ombudsman/Regional Office of the RBI.

Benefits:

• The application improves transparency by keeping the complainants informed

through auto-generated acknowledgements and enabling them to track the status of

their complaints and file appeals online against the decisions of the Ombudsmen,

where applicable.

• Complainants can also voluntarily share feedback on their experience in obtaining

redressal.

7. Consider the following statements regarding Lymphatic filariasis

1. It is caused by infection with parasitic worms living in the lymphatic system.

2. It cause abnormal enlargement of body parts, and leading to severe disability and

social stigmatization of those affected.

Which of the statements given above is/are correct?

(a) 1 only

(b) 2 only

(c) Both 1 and 2

(d) Neither 1 nor 2

Page 9: SIMPLYFYING IAS EXAM PREPARATION - INSIGHTSIAS · Insta 75 Days Revision Plan for UPSC Civil Services Prelims – 2020 This document is the compilation of 100 questions that are part

INSTA 75 Days REVISION PLAN for Prelims 2020 - InstaTests

www.insightsonindia.com 7 Insights IAS

Solution: C

• Also called as elephantiasis, it is caused by infection with parasitic worms living in the

lymphatic system.

• The larval stages of the parasite (microfilaria) circulate in the blood and are

transmitted from person to person by mosquitoes.

• It may Cause abnormal enlargement of body parts, and leading to severe disability

and social stigmatization of those affected.

• Lymphatic filariasis poses a grave threat to India. Over 40% of worldwide cases are

found in India.

8. Consider the following statements regarding National Investment and Infrastructure

Fund (NIIF)

1. It an investment vehicle for funding commercially viable green field, brown field

and stalled infrastructure projects.

2. The Indian government is investing 51% and the rest of the corpus is to be raised

from third-party investors.

Which of the statements given above is/are correct?

(a) 1 only

(b) 2 only

(c) Both 1 and 2

(d) Neither 1 nor 2

Solution: A

National Investment and Infrastructure Fund (NIIF)

• Canada’s largest pension fund Canada Pension Plan Investment Board (CPPIB) has

agreed to invest about $600 million in National Investment and Infrastructure Fund

(NIIF) through the NIIF Master Fund.

About NIIF:

• The government had set up the ₹40,000 crore NIIF in 2015 as an investment vehicle

for funding commercially viable green field, brown field and stalled infrastructure

projects.

Page 10: SIMPLYFYING IAS EXAM PREPARATION - INSIGHTSIAS · Insta 75 Days Revision Plan for UPSC Civil Services Prelims – 2020 This document is the compilation of 100 questions that are part

INSTA 75 Days REVISION PLAN for Prelims 2020 - InstaTests

www.insightsonindia.com 8 Insights IAS

• The Indian government is investing 49% and the rest of the corpus is to be raised from

third-party investors such as sovereign wealth funds, insurance and pension funds,

endowments, etc.

• NIIF’s mandate includes investing in areas such as energy, transportation, housing,

water, waste management and other infrastructure-related sectors in India.

• NIIF currently manages three funds each with its distinctive investment mandate. The

funds are registered as Alternative Investment Fund (AIF) with the Securities and

Exchange Board of India (SEBI).

The three funds are:

• Master Fund: Is an infrastructure fund with the objective of primarily investing in

operating assets in the core infrastructure sectors such as roads, ports, airports, power

etc.

• Fund of Funds: Managed by fund managers who have good track records in

infrastructure and associated sectors in India. Some of the sectors of focus include

Green Infrastructure, Mid-Income & Affordable Housing, Infrastructure services and

allied sectors.

• Strategic Investment Fund: Is registered as an Alternative Investment Fund II under

SEBI in India. The objective is to invest largely in equity and equity-linked instruments.

It will focus on green field and brown field investments in the core infrastructure

sectors.

9. Consider the following statements regarding Laffer Curve

1. The Laffer Curve describes the relationship between tax rates and total tax

revenue.

2. If taxes are too high along the Laffer Curve, then they will increase total tax

revenue.

Which of the statement above is/are correct?

(a) 1 only

(b) 2 only

(c) Both 1 and 2

(d) Neither 1 nor 2

Solution: A

• The Laffer Curve describes the relationship between tax rates and total tax revenue,

with an optimal tax rate that maximizes total government tax revenue.

Page 11: SIMPLYFYING IAS EXAM PREPARATION - INSIGHTSIAS · Insta 75 Days Revision Plan for UPSC Civil Services Prelims – 2020 This document is the compilation of 100 questions that are part

INSTA 75 Days REVISION PLAN for Prelims 2020 - InstaTests

www.insightsonindia.com 9 Insights IAS

• If taxes are too high along the Laffer Curve, then they will discourage the taxed

activities, such as work and investment, enough to actually reduce total tax revenue.

In this case, cutting tax rates will both stimulate economic incentives and increase tax

revenue.

• The Laffer Curve was used as a basis for tax cuts in the 1980’s with apparent success,

but criticized on practical grounds on the basis of its simplistic assumptions, and on

economic grounds that increasing government revenue might not always be optimal.

• The Laffer Curve is based on the economic idea that people will adjust their behavior

in the face of the incentives created by income tax rates. Higher income tax rates

decrease the incentive to work and invest compared lower rates. If this effect is large

enough, it means that at some tax rate, and further increase in the rate will actually

lead to decrease in total tax revenue. For every type of tax, there is a threshold rate

above which the incentive to produce more diminishes, thereby reducing the amount

of revenue the government receives.

10. Consider the following statements regarding Electoral bond scheme

1. An electoral bond is like a promissory note that can be bought by any Indian

citizen or company incorporated in India from select branches of State Bank of

India.

2. The bonds will be issued in multiples of Rs 1,000, Rs 10,000, Rs 100,000 and Rs 1

crore (the range of a bond is between Rs 1,000 to Rs 1 crore).

3. The electoral bond will be valid only for thirty days.

Which of the statements given above is/are correct?

(a) 1 and 2 only

(b) 1 and 3 only

(c) 3 only

(d) 1, 2 and 3

Solution: A

• An electoral bond is like a promissory note that can be bought by any Indian citizen

or company incorporated in India from select branches of State Bank of India. The

citizen or corporate can then donate the same to any eligible political party of his/her

choice. The bonds are similar to bank notes that are payable to the bearer on demand

and are free of interest. An individual or party will be allowed to purchase these bonds

digitally or through cheque.

• Using electoral bonds is quite simple. The bonds will be issued in multiples of Rs 1,000,

Rs 10,000, Rs 100,000 and Rs 1 crore (the range of a bond is between Rs 1,000 to Rs 1

Page 12: SIMPLYFYING IAS EXAM PREPARATION - INSIGHTSIAS · Insta 75 Days Revision Plan for UPSC Civil Services Prelims – 2020 This document is the compilation of 100 questions that are part

INSTA 75 Days REVISION PLAN for Prelims 2020 - InstaTests

www.insightsonindia.com 10 Insights IAS

crore). These will be available at some branches of SBI. A donor with a KYC-compliant

account can purchase the bonds and can then donate them to the party or individual

of their choice. Now, the receiver can encash the bonds through the party’s verified

account.

• The electoral bond will be valid only for fifteen days.

https://www.business-standard.com/about/what-is-electoral-bond

11. Consider the following statements regarding Cash Reserve Ratio

1. It refers to the fraction of the total Net Demand and Time Liabilities (NDTL) of a

Bank, that it has to maintain as cash deposit with the Reserve Bank of India (RBI).

2. The banks are not paid any interest on behalf of the RBI for parking the required

cash.

3. Regional rural banks and co-operative banks are outside the purview of CRR.

Which of the statements given above is/are correct?

(a) 1 and 2 only

(b) 2 and 3 only

(c) 1 and 3 only

(d) 1, 2 and 3

Solution: A

• Cash Reserve Ratio refers to the fraction of the total Net Demand and Time Liabilities

(NDTL) of a Scheduled Commercial Bank held in India, that it has to maintain as cash

deposit with the Reserve Bank of India (RBI). The requirement applies uniformly to all

banks in the country irrespective of an individual bank’s financial situation or size. In

contrast, certain countries e.g. China stipulates separate reserve requirements for

‘large’ and ‘small’ banks.

• As per the RBI Act 1934, all Scheduled Commercial Banks (that includes public and

private sector banks, foreign banks, regional rural banks and co-operative banks) are

required to maintain a cash balance on average with the RBI on a fortnightly basis to

cater to the CRR requirement. Non-Bank Financial Corporations (NBFCs) are outside

the purview of this reserve requirement. Act also authorizes RBI to stipulate an

additional or incremental CRR, which, however, has not been put in place by RBI.

• Banks have to maintain 100 percent CRR on an average basis during the fortnight.

That is, it is not necessary that on all days CRR has to be at 100%. With effect from

December 28, 2002 all banks were required to maintain a minimum of 70 per cent of

the required average daily CRR on all days of the fortnight. Later, with effect from the

Page 13: SIMPLYFYING IAS EXAM PREPARATION - INSIGHTSIAS · Insta 75 Days Revision Plan for UPSC Civil Services Prelims – 2020 This document is the compilation of 100 questions that are part

INSTA 75 Days REVISION PLAN for Prelims 2020 - InstaTests

www.insightsonindia.com 11 Insights IAS

fortnight beginning September 21, 2013 entities were required to maintain minimum

CRR balances up to 95 per cent of the average daily required reserves for a reporting

fortnight on all days of the fortnight. This was later reduced to 90 per cent with effect

from the fortnight beginning April 16, 2016.

• Traditionally, the amount held to cater to the CRR requirement was stipulated to be

no lower than 3 percent and no higher than 20 percent of the total NDTL held in India.

However, the RBI (amendment) Act, 2006 provides for removal of the floor and ceiling

with respect to setting the CRR and authorizes the RBI to set the ratio in keeping with

the broad objective of maintaining monetary stability in the economy.

• Presently, banks are not paid any interest on behalf of the RBI for parking the required

cash. If a bank fails to meet its required reserve requirements, the RBI is empowered

to impose a penalty by charging a penal interest rate.

12. Consider the following statements regarding Fixed Income Money Market and

Derivatives Association of India

1. It is an association of Scheduled Commercial Banks, Public Financial Institutions,

Primary Dealers and Insurance Companies.

2. FIMMDA is a voluntary market body for the bond, money and derivatives markets.

Which of the statements given above is/are correct?

(a) 1 only

(b) 2 only

(c) Both 1 and 2

(d) Neither 1 nor 2

Solution: C

• The Fixed Income Money Market and Derivatives Association of India (FIMMDA), an

association of Scheduled Commercial Banks, Public Financial Institutions, Primary

Dealers and Insurance Companies was incorporated as a Company under section 25

of the Companies Act,1956 on June 3, 1998. FIMMDA is a voluntary market body for

the bond, money and derivatives markets. FIMMDA has members representing all

major institutional segments of the market. The membership includes Nationalized

Banks such as State Bank of India, its associate banks and other nationalized banks;

Private sector banks such as ICICI Bank, HDFC Bank; Foreign Banks such as Bank of

America, Citibank, Financial institutions such as IDFC, EXIM Bank, NABARD, Insurance

Companies like Life Insurance Corporation of India (LIC), ICICI Prudential Life Insurance

Company, Birla Sun Life Insurance Company and all Primary Dealers.

Page 14: SIMPLYFYING IAS EXAM PREPARATION - INSIGHTSIAS · Insta 75 Days Revision Plan for UPSC Civil Services Prelims – 2020 This document is the compilation of 100 questions that are part

INSTA 75 Days REVISION PLAN for Prelims 2020 - InstaTests

www.insightsonindia.com 12 Insights IAS

• FIMMDA represents market participants and aids the development of the bond,

money and derivatives markets. It acts as an interface with the regulators on various

issues that impact the functioning of these markets. FIMMDA also plays a constructive

role in the evolution of best market practices by its members so that the market as a

whole operates transparently as well as efficiently.

13. Glyphosate, often seen in the news, is related to

(a) Multidrug Resistance Tuberculosis

(b) Bio-chemical agent to remove oil spills

(c) Weedicide

(d) None of the above

Solution: C

• Glyphosate’s increase resulted from the rapid adoption of genetically engineered

crops and no-till farming practices, both of which incorporate glyphosate for weed

control (Gianessi and Reigner, 2006).

• The first triazine herbicide, atrazine, was discovered by J.R. Geigy, Ltd., in Switzerland

(LeBaron et al., 2008). It was first registered in the United States in 1958. Atrazine has

a range of trade names such as Marksman, Coyote, Atrazina, Atrazol, and Vectal (PAN,

2002) and is estimated to be the second most heavily used herbicide in the United

States at 35,000 metric tons (77 million lbs) (Gianessi and Reigner, 2006).

https://www.sciencedirect.com/topics/earth-and-planetary-sciences/glyphosate

14. Consider the following statements regarding Wholesale Price Index (WPI)

1. Primary Articles forms the major component of WPI.

2. It is measured on year-on-year basis i.e., rate of change in price level in a given

month vis a vis corresponding month of last year.

Which of the statements given above is/are correct?

(a) 1 only

(b) 2 only

(c) Both 1 and 2

(d) Neither 1 nor 2

Page 15: SIMPLYFYING IAS EXAM PREPARATION - INSIGHTSIAS · Insta 75 Days Revision Plan for UPSC Civil Services Prelims – 2020 This document is the compilation of 100 questions that are part

INSTA 75 Days REVISION PLAN for Prelims 2020 - InstaTests

www.insightsonindia.com 13 Insights IAS

Solution: B

In general, reflects the rate of change in prices of all goods and services in an economy over

a period of time. Every country has its own set of commodity basket to track inflation. While

some countries use Wholesale Price Index (WPI) as their official measure of inflation and

some others use the Consumer Price Index (CPI). The International Monetary Fund (IMF)

statistics reveals that, while 24 countries use WPI as the official measure to track inflation,

157 countries use CPI. Conceptually these two measures of inflation stress different stages of

price realization as well as composition: while WPI measures the change in price level at

wholesale market, CPI measures the change in price level at retail level.

In India, headline inflation is measured through the WPI (the latest base year 2011-12) – which

consists of 697 commodities (services are not included in WPI in India). It is measured on year-

on-year basis i.e., rate of change in price level in a given month vis a vis corresponding month

of last year. This is also known as point to point inflation.

Apart from WPI, CPI is also computed to capture inflation in India. In particular, four

categories of CPI are computed – for Industrial Workers (CPI-IW), Urban Non-Manual

Employees (CPI-UNME), Agricultural Labourers (CPI-AL) and Rural Labourers (CPI-RL).

However, WPI is considered as the preferred measure of headline inflation due to its wider

coverage. To overcome this lacuna, the Central Statistical Organization (on 18th February

2011) has introduced a new series of CPI (with 2010=100 as the base year), which would be

calculated for all-India as well as States/UTs – with separate categorization for rural, urban

and combined (rural + urban).

Who publishes WPI in India and what does it show?

• Analysts use the numbers to track the supply and demand dynamics in industry,

manufacturing and construction. The numbers are released by the Economic Advisor

in the Ministry of Commerce and Industry. An upward surge in the WPI print indicates

inflationary pressure in the economy and vice versa. The quantum of rise in the WPI

month-after-month is used to measure the level of wholesale inflation in the

economy.

What is the difference between WPI and CPI inflation?

• While WPI keeps track of the wholesale price of goods, the CPI measures the average

price that households pay for a basket of different goods and services. Even as the WPI

is used as a key measure of inflation in some economies, the RBI no longer uses it for

policy purposes, including setting repo rates. The central bank currently uses CPI or

retail inflation as a key measure of inflation to set the monetary and credit policy.

New series of WPI

• With an aim to align the index with the base year of other important economic

indicators such as GDP and IIP, the base year was updated to 2011-12 from 2004-05

for the new series of Wholesale Price Index (WPI), effective from April 2017.

Page 16: SIMPLYFYING IAS EXAM PREPARATION - INSIGHTSIAS · Insta 75 Days Revision Plan for UPSC Civil Services Prelims – 2020 This document is the compilation of 100 questions that are part

INSTA 75 Days REVISION PLAN for Prelims 2020 - InstaTests

www.insightsonindia.com 14 Insights IAS

How do you calculate Wholesale Price Index?

• The monthly WPI number shows the average price changes of goods usually expressed

in ratios or percentages.

• The index is based on the wholesale prices of a few relevant commodities available.

• The commodities are chosen based on their significance in the region. These represent

different strata of the economy and are expected to provide a comprehensive WPI

value.

• The advanced base year 2011-12 adopted recently uses 697 items.

15. Consider the following statements regarding India Brand Equity Foundation (IBEF)

1. It is a Trust established by the Confederation of Indian Industry (CII).

2. Its primary objective is to promote and create international awareness of the

Made in India label.

Which of the statement above is/are correct?

(a) 1 only

(b) 2 only

(c) Both 1 and 2

(d) Neither 1 nor 2

Solution: B

India Brand Equity Foundation (IBEF)

• India Brand Equity Foundation (IBEF) is a Trust established by the Department of

Commerce, Ministry of Commerce and Industry, Government of India. IBEF’s primary

objective is to promote and create international awareness of the Made in India label

Page 17: SIMPLYFYING IAS EXAM PREPARATION - INSIGHTSIAS · Insta 75 Days Revision Plan for UPSC Civil Services Prelims – 2020 This document is the compilation of 100 questions that are part

INSTA 75 Days REVISION PLAN for Prelims 2020 - InstaTests

www.insightsonindia.com 15 Insights IAS

in markets overseas and to facilitate dissemination of knowledge of Indian products

and services. Towards this objective, IBEF works closely with stakeholders across

government and industry

• India, today, is well established as a credible business partner, preferred investment

destination, rapidly growing market, provider of quality services and manufactured

products; and, stands on the threshold years of unprecedented growth.

• India’s Talent, Markets, Growth and Opportunity drive Brand India.

• http://www.ibef.org is a knowledge centre for global investors, international policy-

makers and world media seeking updated, accurate and comprehensive information

on the Indian economy, states and sectors. IBEF regularly tracks government

announcements in policy, foreign investment, macroeconomic indicators and

business trends.

IBEF works with a network of stakeholders – domestic and international – to promote Brand

India.

16. Consider the following pairs of Martial arts and the states they are mainly associated

with:

Martial Arts States 1. Silambam : Odisha 2. Gatka : Maharashtra 3. Huyen langlon : Arunachal Pradesh

Which of the pairs given above is/are correctly matched?

(a) 1 and 3 only

(b) 2 and 3 only

(c) 1 only

(d) None

Solution: D

• Silambam is a weapon-based Indian martial art originating in modern-day Tamil

Nadu in the Indian subcontinent and is estimated to have originated in approximately

1000 BCE. This ancient fighting style is mentioned in Tamil Sangam literature 400 BCE.

• Gatka is the name of an Indian martial art associated with the Sikhs of the Punjab

and the Tanoli and Gujjar communities of the mountainous regions of northern

Pakistan who practice an early variant of the martial art. It is a style of stick-fighting,

with wooden sticks intended to simulate swords.

• Huyen langlon is an Indian martial art from Manipur. In the Meitei language, huyen

means war while langlon or langlong can mean net, knowledge or art. Huyen langlon

Page 18: SIMPLYFYING IAS EXAM PREPARATION - INSIGHTSIAS · Insta 75 Days Revision Plan for UPSC Civil Services Prelims – 2020 This document is the compilation of 100 questions that are part

INSTA 75 Days REVISION PLAN for Prelims 2020 - InstaTests

www.insightsonindia.com 16 Insights IAS

consists of two main components: thang-ta and sarit sarak. The primary weapons of

huyen langlon are the thang and ta.

17. Consider the following statements regarding Effective Revenue deficit

1. The term Effective Revenue deficit was introduced in the Union Budget 2011-12.

2. Effective Revenue Deficit signifies that amount of capital receipts that are being

used for actual capital expenditure of the Government.

3. Since its inception, Year on year the effective revenue deficit is decreasing.

Which of the statements given above is/are correct?

(a) 1 and 2 only

(b) 2 only

(c) 1 and 3 only

(d) 1, 2 and 3

Solution: A

• Effective Revenue deficit is a new term introduced in the Union Budget 2011-12.

While revenue deficit is the difference between revenue receipts and revenue

expenditure, the present accounting system includes all grants from the Union

Government to the state governments/Union territories/other bodies as revenue

expenditure, even if they are used to create assets. Such assets created by the sub-

national governments/bodies are owned by them and not by the Union Government.

Nevertheless they do result in the creation of durable assets.

• According to the Finance Ministry, such revenue expenditures contribute to the

growth in the economy and therefore, should not be treated as unproductive in

nature. In the Union Budget (2011-12) a new methodology has been introduced to

capture the ‘effective revenue deficit’, which excludes those revenue expenditures

(or transfers) in the form of grants for creation of capital assets. If this methodology is

taken into account, the effective revenue deficit (revised estimates) for 2010-11 is

only 2.3 per cent as against the revenue deficit of 3.4 per cent of GDP. The effective

revenue deficit for 2011-12 is projected at 1.8 per cent as against the revenue deficit

estimates of 3.4 per cent.

• It may be noted that even though some grants may be allocated towards the creation

of assets, financial allocation does not always result in physical outcomes.

• Grants for creation of capital assets, as a concept, was introduced in the FRBM Act

through the amendment in 2012. The Act defines grants for creation of capital assets

as grants-in-aid given by the Central Government to state governments, autonomous

Page 19: SIMPLYFYING IAS EXAM PREPARATION - INSIGHTSIAS · Insta 75 Days Revision Plan for UPSC Civil Services Prelims – 2020 This document is the compilation of 100 questions that are part

INSTA 75 Days REVISION PLAN for Prelims 2020 - InstaTests

www.insightsonindia.com 17 Insights IAS

bodies, local bodies and other scheme implementing agencies for creation of capital

assets which are owned by these entities.

• In short, Effective Revenue Deficit is the difference between revenue deficit and grants

for creation of capital assets. Effective Revenue Deficit signifies that amount of capital

receipts that are being used for actual consumption expenditure of the Government.

18. Consider the following statements regarding National Strategy for Financial Inclusion

1. It was prepared by Ministry of Finance.

2. Every village to have access to a formal financial service provider within a

reasonable distance of 15 KM radius.

Which of the statements given above is/are correct?

(a) 1 only

(b) 2 only

(c) Both 1 and 2

(d) Neither 1 nor 2

Solution: D

The National Strategy for Financial Inclusion 2019-2024 sets forth the vision and key

objectives of the financial inclusion policies in India to help expand and sustain the financial

inclusion process at the national level through a broad convergence of action involving all the

stakeholders in the financial sector.

The strategy aims to provide access to formal financial services in an affordable manner,

broadening & deepening financial inclusion and promoting financial literacy & consumer

protection.

The National Strategy for Financial Inclusion for India 2019-2024 has been prepared by RBI

under the aegis of the Financial Inclusion Advisory Committee and is based on the inputs and

suggestions from Government of India, other Financial Sector Regulators viz., Securities

Exchange Board of India (SEBI), Insurance Regulatory and Development Authority of India

(IRDAI) and Pension Fund Regulatory and Development Authority of India (PFRDA).

To achieve the vision of ensuring access to an array of basic formal financial services, a set of

guiding objectives have been formulated with special relevance in the Indian context.

• Universal Access to Financial Services: Every village to have access to a formal

financial service provider within a reasonable distance of 5 KM radius. The customers

may be on boarded through an easy and hassle-free digital process and processes

should be geared towards a less-paper ecosystem.

Page 20: SIMPLYFYING IAS EXAM PREPARATION - INSIGHTSIAS · Insta 75 Days Revision Plan for UPSC Civil Services Prelims – 2020 This document is the compilation of 100 questions that are part

INSTA 75 Days REVISION PLAN for Prelims 2020 - InstaTests

www.insightsonindia.com 18 Insights IAS

• Providing Basic Bouquet of Financial Services: Every adult who is willing and eligible

needs to be provided with a basic bouquet of financial services that include a Basic

Savings Bank Deposit Account, credit, a micro life and non-life insurance product, a

pension product and a suitable investment product.

• Access to Livelihood and Skill Development: The new entrant to the financial system,

if eligible and willing to undergo any livelihood/ skill development programme, may

be given the relevant information about the ongoing Government livelihood

programmes thus helping them to augment their skills and engage in meaningful

economic activity and improve income generation.

• Customer Protection and Grievance Redressal: Customers shall be made aware of the

recourses available for resolution of their grievances. About storing and sharing of

customer’s biometric and demographic data, adequate safeguards need to be ensured

to protect the customer’s Right to Privacy.

• Effective Co-ordination: There needs to be a focused and continuous coordination

between the key stakeholders viz. Government, the Regulators, financial service

providers, Telecom Service Regulators, Skills Training institutes etc. to make sure that

the customers are able to use the services in a sustained manner. The focus shall be

to consolidate gains from previous efforts through focus on improvement of quality

of service of last mile delivery viz., capacity building of Business Correspondents,

creating payments system ecosystems at village levels to deepen the culture of digital

finance leading to ease of use and delivery.

19. Idu-Mishmi tribe often seen in the news, is found in

(a) Assam

(b) Kashmir

(c) Arunachal Pradesh

(d) Manipur

Solution: C

• Idu-Mishmi tribe is the lone inhabitant tribe of Dibang Valley district. They are of

mongoloid race and have distinctive dialect of Tibeto-Burma language. Idu-Mishmi

tribe can be distinctively identified among other tribal groups of Arunachal Pradesh by

their typical hairstyle, distinctive customs and artistic pattern embedded on their

clothes. This tribe still maintains deep rooted aesthetic values in their day to day life

with great pride and honour.

Page 21: SIMPLYFYING IAS EXAM PREPARATION - INSIGHTSIAS · Insta 75 Days Revision Plan for UPSC Civil Services Prelims – 2020 This document is the compilation of 100 questions that are part

INSTA 75 Days REVISION PLAN for Prelims 2020 - InstaTests

www.insightsonindia.com 19 Insights IAS

20. Consider the following statements regarding National Credit Guarantee Trustee

Company Ltd [NCGTC]

1. It was set up by the Department of Financial Services, Ministry of Finance with a

paid-up capital of ₹100 crore.

2. It acts as a common trustee company to manage and operate various credit

guarantee trust funds.

Which of the statement above is/are correct?

(a) 1 only

(b) 2 only

(c) Both 1 and 2

(d) Neither 1 nor 2

Solution: B

• Subsequent to the Central Budget announcements to set up various credit guarantee

funds, a common trustee company in the name and style of National Credit

Guarantee Trustee Company Ltd [NCGTC] was set up by the Department of Financial

Services, Ministry of Finance, Government of India to, inter alia, act as a common

trustee company to manage and operate various credit guarantee trust funds.

• NCGTC was incorporated under the Indian Companies Act, 1956 on March 28, 2014

with a paid-up capital of ₹10 crore,

Current Trust Funds under the trusteeship management of NCGTC:

1) Credit Guarantee Fund for Skill Development (CGFSD)

• Guarantees for Skill Development Loans by the member banks of IBA up to ₹ 1.5 lakh

extended without collateral or third-party guarantee and the fund has a Target of 10-

20 lakh loans to be guaranteed in a year.

2) Credit Guarantee Fund for Education loans (CGFEL)

• Guarantees for Education Loans by the member banks of IBA up to ₹ 7.5 lakh extended

without collateral or third-party guarantee and the fund has a Target of 10 lakh loans

to be guaranteed in a year.

3) Credit Guarantee Fund for Factoring (CGFF)

• Guarantees for domestic factored debts of MSMEs.

Page 22: SIMPLYFYING IAS EXAM PREPARATION - INSIGHTSIAS · Insta 75 Days Revision Plan for UPSC Civil Services Prelims – 2020 This document is the compilation of 100 questions that are part

INSTA 75 Days REVISION PLAN for Prelims 2020 - InstaTests

www.insightsonindia.com 20 Insights IAS

4) Credit Guarantee Fund for Micro Units (CGFMU)

• Guarantees for loans up to the specified limit(currently ₹ 10Lakh) sanctioned by Banks

/ NBFCs / MFIs / other financial intermediaries engaged in providing credit facilities to

eligible micro units. Further, Overdraft loan amount of ₹ 5,000/- sanctioned under

PMJDY accounts shall also be eligible to be covered under Credit guarantee Fund.

5) Credit Guarantee Fund for Standup India (CGFSI)

• Guarantees for credit facilities of over ₹ 10 lakh & upto ₹ 100 lakh sanctioned by the

eligible lending institutions, under the Stand Up India Scheme(SC/ST/Women for

setting up Greenfield enterprises).

21. Consider the following statements regarding Koundinya Wildlife Sanctuary

1. It is the only sanctuary in Telangana with a population of Asian elephants.

2. It is covered by southern tropical dry deciduous and thorn forests.

Which of the statements given above is/are correct?

(a) 1 only

(b) 2 only

(c) Both 1 and 2

(d) Neither 1 nor 2

Solution: B

Koundinya Wildlife Sanctuary

• Kaundinya Wildlife Sanctuary is a wildlife sanctuary and an elephant reserve situated

in Andhra Pradesh, India. It is the only sanctuary in Andhra Pradesh with a

population of Asian elephants, which migrated after 200 years from neighbouring

regions.

• The sanctuary is covered by southern tropical dry deciduous and thorn forests. Some

of the important flora consists of Albiziaamara, Acacia, Lagerstroemia, Ficus,

bamboo, and a species which is a regeneration of Santalum album.

• The sanctuary is situated in region where the Kolar Plateau ends and slopes down into

the plains of Tamil Nadu creating many valleys and ghats

• The sanctuary is primarily an elephant reserve and is home to about 78 Indian

elephants.

Page 23: SIMPLYFYING IAS EXAM PREPARATION - INSIGHTSIAS · Insta 75 Days Revision Plan for UPSC Civil Services Prelims – 2020 This document is the compilation of 100 questions that are part

INSTA 75 Days REVISION PLAN for Prelims 2020 - InstaTests

www.insightsonindia.com 21 Insights IAS

22. Consider the following statements regarding Purchasing Managers’ Index (PMI)

1. It is released by Reserve Bank of India.

2. It is an indicator of business activity, both in the manufacturing and services

sectors.

3. A figure above 50 denotes expansion in business activity. Anything below 50

denotes contraction.

Which of the statement above is/are correct?

(a) 2 only

(b) 2 and 3 only

(c) 1 and 3 only

(d) 1, 2 and 3

Solution: B

Purchasing Managers’ Index (PMI) by Japanese firm Nikkei

• It is an indicator of business activity – both in the manufacturing and services sectors.

• The purpose of the PMI is to provide information about current business conditions

to company decision makers, analysts and purchasing managers.

• It is a survey-based measures that asks the respondents about changes in their

perception of some key business variables from the month before.

• A figure above 50 denotes expansion in business activity. Anything below 50 denotes

contraction.

23. Consider the following statements regarding Interpol’s Red Notices

1. It has been issued by Interpol general secretariat

2. It is an international arrest warrant

3. Interpol can compel any member country to arrest an individual subject to red

notice

Which of the statements given above is/are correct?

(a) 1 and 3 only

(b) 2 and 3 only

(c) 1 only

(d) 1, 2 and 3

Page 24: SIMPLYFYING IAS EXAM PREPARATION - INSIGHTSIAS · Insta 75 Days Revision Plan for UPSC Civil Services Prelims – 2020 This document is the compilation of 100 questions that are part

INSTA 75 Days REVISION PLAN for Prelims 2020 - InstaTests

www.insightsonindia.com 22 Insights IAS

Solution: C

Red Notice:

• A Red Notice is a request to law enforcement worldwide to locate and provisionally

arrest a person pending extradition, surrender, or similar legal action.

It contains two main types of information:

• Information to identify the wanted person, such as their name, date of birth,

nationality, hair and eye colour, photographs and fingerprints if available.

• Information related to the crime they are wanted for, which can typically be murder,

rape, child abuse or armed robbery.

• Red Notices are published by INTERPOL at the request of a member country, and

must comply with INTERPOL’s Constitution and Rules.

• A Red Notice is not an international arrest warrant

• Interpol cannot compel any member country to arrest an individual subject to red

notice.

24. Consider the following statements regarding Ways and Means Advances (WMA)

1. They are temporary loan facilities provided by RBI to the government to enable it

to meet temporary mismatches between revenue and expenditure.

2. The rate of interest is the same as the repo rate, while the tenure is three months.

Which of the statement above is/are correct?

(a) 1 only

(b) 2 only

(c) Both 1 and 2

(d) Neither 1 nor 2

Solution: C

• The Reserve Bank of India (RBI) has announced a 60% increase in the Ways and

Means Advances (WMA) limit of state governments over and above the level as on

March 31, with a view to enabling them “to undertake COVID-19 containment and

mitigation efforts” and “to better plan their market borrowings”.

Page 25: SIMPLYFYING IAS EXAM PREPARATION - INSIGHTSIAS · Insta 75 Days Revision Plan for UPSC Civil Services Prelims – 2020 This document is the compilation of 100 questions that are part

INSTA 75 Days REVISION PLAN for Prelims 2020 - InstaTests

www.insightsonindia.com 23 Insights IAS

Significance of this move:

• The increased limit comes at a time when government expenditure is expected to rise

as it battles the fallout of a spreading Coronavirus. The availability of these funds will

give government some room to undertake short term expenditure over and above its

long term market borrowings.

What are Ways and Means Advances?

• They are temporary loan facilities provided by RBI to the government to enable it to

meet temporary mismatches between revenue and expenditure.

• The government makes an interest payment to the central bank when it borrows

money.

• The rate of interest is the same as the repo rate, while the tenure is three months.

• The limits for WMA are mutually decided by the RBI and the Government of India.

• They aren’t a source of finance per se. Section 17(5) of the RBI Act, 1934 authorises

the central bank to lend to the Centre and state governments subject to their being

repayable “not later than three months from the date of the making of the advance”.

Background:

• The WMA scheme for the Central Government was introduced on April 1, 1997, after

putting an end to the four-decade old system of ad-hoc (temporary) Treasury Bills to

finance the Central Government deficit.

What if the government needs extra money for extra time?

• When the WMA limit is crossed the government takes recourse to overdrafts, which

are not allowed beyond 10 consecutive working days.

• The interest rate on overdrafts would be 2 percent more than the repo rate.

Types of WMA:

There are two types of Ways and Means Advances — normal and special.

1. Special WMA or Special Drawing Facility is provided against the collateral of the

government securities held by the state. After the state has exhausted the limit of SDF,

it gets normal WMA. The interest rate for SDF is one percentage point less than the

repo rate.

2. The number of loans under normal WMA is based on a three-year average of actual

revenue and capital expenditure of the state.

What are the existing WMA limits and overdraft conditions?

• For the Centre, the WMA limit during the first half of 2020-21 (April-September) has

been fixed at Rs 120,000 crore. This is 60% higher than the Rs 75,000 crore limit for

the same period of 2019-20. The limit for the second half of the last fiscal (October-

March) was Rs 35,000 crore.

Page 26: SIMPLYFYING IAS EXAM PREPARATION - INSIGHTSIAS · Insta 75 Days Revision Plan for UPSC Civil Services Prelims – 2020 This document is the compilation of 100 questions that are part

INSTA 75 Days REVISION PLAN for Prelims 2020 - InstaTests

www.insightsonindia.com 24 Insights IAS

• For the states, the aggregate WMA limit was Rs 32,225 crore till March 31, 2020. On

April 1, the RBI announced a 30% hike in this limit, which has now been enhanced to

60%, taking it to Rs 51,560 crore. The higher limit will be valid till September 30.

• The central bank, on April 7, also extended the period for which a state can be in

overdraft from 14 to 21 consecutive working days, and from 36 to 50 working days

during a quarter.

25. Consider the following statements regarding Institute of Chartered Accountants of India

1. It is a statutory body

2. It functions under the administrative control of the Ministry of Corporate Affairs

3. It is aimed at regulating the profession of Chartered Accountancy in the country.

Which of the statements given above is/are correct?

(a) 2 and 3 only

(b) 1 only

(c) 1 and 2 only

(d) 1, 2 and 3

Solution: D

Institute of Chartered Accountants of India (ICAI)

• The Institute of Chartered Accountants of India (ICAI) is a statutory body established

by an Act of Parliament of India, ‘The Chartered Accountants Act, and 1949′, to

regulate the profession of Chartered Accountancy in India.

• ICAI is the second largest professional Accounting & Finance body in the world.

• ICAI is the only licensing cum regulating body of the financial audit and accountancy

profession in India.

• It recommends the accounting standards to be followed by companies in India to

National Advisory Committee on Accounting Standards (NACAS).

• ICAI is solely responsible for setting the Standards on Auditing (SAs) to be followed in

the audit of financial statements in India.

• ICAI is one of the founder members of the International Federation of Accountants

(IFAC), South Asian Federation of Accountants (SAFA), and Confederation of Asian

and Pacific Accountants (CAPA).

• The Institute functions under the administrative control of the Ministry of Corporate

Affairs.

Page 27: SIMPLYFYING IAS EXAM PREPARATION - INSIGHTSIAS · Insta 75 Days Revision Plan for UPSC Civil Services Prelims – 2020 This document is the compilation of 100 questions that are part

INSTA 75 Days REVISION PLAN for Prelims 2020 - InstaTests

www.insightsonindia.com 25 Insights IAS

DAY – 66 (InstaTest-66)

26. Consider the following statements regarding Armed Forces (Special Powers) Act

(AFSPA).

1. The Central Government, or the Governor of the State or administrator of the

Union Territory can declare the whole or part of the State or Union Territory as a

disturbed area.

2. Presently, it is operational in the entire States of Assam, Nagaland and Manipur

only

3. Armed can use force or even open fire after giving due warning if they feel a

person is in contravention of the law

Which of the statements given above is/are correct?

(a) 1 and 3 only

(b) 2 and 3 only

(c) 1 only

(d) 1, 2 and 3

Solution: A

Armed Forces (Special Powers) Act (AFSPA) gives armed forces the power to maintain public

order in “disturbed areas”. They have the authority to prohibit a gathering of five or more

persons in an area, can use force or even open fire after giving due warning if they feel a

person is in contravention of the law

• The Central Government, or the Governor of the State or administrator of the Union

Territory can declare the whole or part of the State or Union Territory as a disturbed

area

• Presently, AFSPA, 1958, is operational in the entire States of Assam, Nagaland,

Manipur (except Imphal Municipal area), three districts namely Tirap, Changlang and

Longding of Arunachal Pradesh and the areas falling within the jurisdiction of the

eight police stations in the districts of Arunachal Pradesh, bordering the State of

Assam.

• The notification declaring Manipur and Assam as “Disturbed Areas’ has been issued

by the State governments. For Nagaland, the notification is issued by the MHA.

• It is a suo-motto declaration can be made by the Central government, however, it is

desirable that the state government should be consulted by the central government

before making the declaration.

Page 28: SIMPLYFYING IAS EXAM PREPARATION - INSIGHTSIAS · Insta 75 Days Revision Plan for UPSC Civil Services Prelims – 2020 This document is the compilation of 100 questions that are part

INSTA 75 Days REVISION PLAN for Prelims 2020 - InstaTests

www.insightsonindia.com 26 Insights IAS

https://www.thehindu.com/news/national/other-states/afspa-extended-in-

nagaland/article30434509.ece

27. Consider the following statements regarding National Sports Development Fund (NSDF)

1. It was established in 1998 under the Charitable Endowments Act 1890.

2. The purpose of creation is to impart momentum and flexibility to assisting the

cause of sports

3. The Fund is managed by a Council and Union Minister in charge of Youth Affairs

and Sports is the Chairperson of the Council.

Which of the statements given above is/are correct?

(a) 1 and 2 only

(b) 2 and 3 only

(c) 1 and 3 only

(d) 1, 2 and 3

Solution: D

NATIONAL SPORTS DEVELOPMENT FUND

• National Sports Development Fund (NSDF) was established in 1998 under the

Charitable Endowments Act 1890;

• It was notified by Government of India in November, 1998;

• Purpose of creation is to impart momentum and flexibility to assisting the cause of

sports;

• The Fund helps sportspersons excel by providing them opportunities to train under

coaches of international repute with technical, scientific and psychological support

and giving them exposure to international competitions;

• The Fund also provides financial assistance for development of infrastructure and

other activities for promotion of sports;

• Role of the Fund is supplementary to the overall policy and activities of the

Department of Sports in achieving excellence in sports.

OBJECTIVES OF NSDF

• To administer and apply the moneys of the Fund for promotion of sports in general

and specific sports disciplines and individual sports persons in particular for achieving

excellence at the National and of International level;

• To impart special training and coaching in relevant sports disciplines to sports persons,

coaches and sports specialists;

Page 29: SIMPLYFYING IAS EXAM PREPARATION - INSIGHTSIAS · Insta 75 Days Revision Plan for UPSC Civil Services Prelims – 2020 This document is the compilation of 100 questions that are part

INSTA 75 Days REVISION PLAN for Prelims 2020 - InstaTests

www.insightsonindia.com 27 Insights IAS

• To construct and maintain infrastructure for promotion of sports and games;

• To supply sports equipments to organizations and individuals for promotion of sports

and games;

• To identify problems and take up research and development studies for providing

support to excellence in sports;

• To promote international cooperation, in particular, exchanges which may promote

the development of sports;

• To provide low-interest or interest-free loans for projects and activities related to any

of the aforesaid objects.

MANAGEMENT AND ADMINISTRATION OF NSDF

Council of NSDF

• The Fund is managed by a Council constituted by the Central Government. Union

Minister in charge of Youth Affairs and Sports is the Chairperson of the Council.

Members of the Council include senior Officers of the Department of Sports, Chairman

& Managing Directors of Private and Public Sector Companies / Corporations,

representatives of Sports Promotion Boards, etc.

• The Council decides all policy matters relating to the Fund.

28. Consider the following statements regarding exchange rate

1. The Real Effective Exchange Rate (REER) of the rupee is a weighted average of

exchange rates before the currencies of India’s major trading partners.

2. When the weight of inflation is adjusted with the REER, we get the Nominal

Effective Exchange Rate (REER) of the rupee.

Which of the statements given above is/are correct?

(a) 1 only

(b) 2 only

(c) Both 1 and 2

(d) Neither 1 nor 2

Solution: D

NEER

• The Nominal Effective Exchange Rate (NEER) of the rupee is a weighted average of

exchange rates before the currencies of India’s major trading partners.

Page 30: SIMPLYFYING IAS EXAM PREPARATION - INSIGHTSIAS · Insta 75 Days Revision Plan for UPSC Civil Services Prelims – 2020 This document is the compilation of 100 questions that are part

INSTA 75 Days REVISION PLAN for Prelims 2020 - InstaTests

www.insightsonindia.com 28 Insights IAS

REER

• When the weight of inflation is adjusted with the NEER, we get the Real Effective

Exchange Rate (REER) of the rupee. Since inflation has been on the higher side in

recent months, the REER of the rupee has been more against it than the NEER.

29. Consider the following statements regarding Skills Build Platform

1. It is skill building programme for migrant workers.

2. It is launched by Ministry of Labour with active co-operation by skill ministry

Which of the statements given above is/are correct?

(a) 1 only

(b) 2 only

(c) Both 1 and 2

(d) Neither 1 nor 2

Solution: D

• Ministry of Skill Development & Entrepreneurship launches Skills Build platform in

Collaboration with IBM.

Key facts:

• Launched by Directorate General of Training (DGT), under the aegis of Ministry of

Skill Development & Entrepreneurship (MSDE).

• As part of the programme, a two-year advanced diploma in IT, networking and cloud

computing, co-created and designed by IBM, will be offered at the Industrial Training

Institutes (ITIs) & National Skill Training Institutes (NSTIs).

• The platform will be extended to train ITI & NSTI faculty on building skills in Artificial

Intelligence (AI).

Significance of the programme:

• The digital platform will provide a personal assessment of the cognitive capabilities

and personality via My Inner Genius to the students.

• They will then learn foundational knowledge about digital technologies, as well as

professional skills such as resume-writing, problem solving and communication.

• Students will also receive recommendations on role-based education for specific jobs

that include technical and professional learning.

• This initiative is part of IBM’s global commitment to create a job-ready workforce and

to build the next generation of skills needed for new collar careers.

Page 31: SIMPLYFYING IAS EXAM PREPARATION - INSIGHTSIAS · Insta 75 Days Revision Plan for UPSC Civil Services Prelims – 2020 This document is the compilation of 100 questions that are part

INSTA 75 Days REVISION PLAN for Prelims 2020 - InstaTests

www.insightsonindia.com 29 Insights IAS

https://www.insightsonindia.com/2019/11/05/skills-build-platform/

30. Consider the following statements regarding Hybrid Annuity Model (HAM)

1. The HAM is a mix between the existing two models – BOT Annuity and EPC.

2. As per the design, the government will contribute to 60% of the project cost in

the first five years through annual payments (annuity).

3. Revenue collection would be the responsibility of the Government.

Which of the statements given above is/are correct?

(a) 1 and 2 only

(b) 3 only

(c) 1 and 3 only

(d) 1, 2 and 3

Solution: C

Hybrid Annuity Model (HAM) has been introduced by the Government to revive PPP (Public

Private Partnership) in highway construction in India.

• At present, three different models –PPP Annuity, PPP Toll and EPC (Engineering,

Procurement and Construction) were followed by the government while adopting

private sector participation.

Features of HAM

• By features the HAM is a mix between the existing two models – BOT Annuity and EPC.

Hence to understand the HAM, the basic features of the existing PPP models are

elucidated first.

a) The Build Operate and Transfer (BOT) Annuity Model

• Under BOT annuity, a developer builds a highway, operates it for a specified duration

and transfers it back to the government. The government starts payment to the

developer after the launch of commercial operation of the project. Payment will be

made on a six month basis.

b) BOT Toll Model

• In this toll based BOT model, a road developer constructs the road and he is allowed

to recover his investment through toll collection. This toll collection will be over a long

period which is nearly 30 years in most cases. There is no government payment to the

developer as he earns his money invested from tolls.

Page 32: SIMPLYFYING IAS EXAM PREPARATION - INSIGHTSIAS · Insta 75 Days Revision Plan for UPSC Civil Services Prelims – 2020 This document is the compilation of 100 questions that are part

INSTA 75 Days REVISION PLAN for Prelims 2020 - InstaTests

www.insightsonindia.com 30 Insights IAS

c) Engineering, Procurement and Construction (EPC) Model

• Under this model, the cost is completely borne by the government. Government

invites bids for engineering knowledge from the private players. Procurement of raw

material and construction costs are met by the government. The private sector’s

participation is minimum and is limited to the provision of engineering expertise. A

difficulty of the model is the high financial burden for the government.

The Hybrid Annuity Model (HAM):

• In India, the new HAM is a mix of BOT Annuity and EPC models. As per the design, the

government will contribute to 40% of the project cost in the first five years through

annual payments (annuity). The remaining payment will be made on the basis of the

assets created and the performance of the developer.

• Here, hybrid annuity means the first 40% payment is made as fixed amount in five

equal installments whereas the remaining 60% is paid as variable annuity amount

after the completion of the project depending upon the value of assets created.

• As the government pays only 40%, during the construction stage, the developer should

find money for the remaining amount.

• Here, he has to raise the remaining 60% in the form of equity or loans.

• There is no toll right for the developer. Under HAM, Revenue collection would be the

responsibility of the National Highways Authority of India (NHAI).

31. Consider the following statements regarding Invest India

1. It is a non-profit venture under the Department for Promotion of Industry and

Internal Trade, Ministry of Commerce and Industry.

2. It is the National Investment Promotion and Facilitation Agency of India and act

as the first point of reference for investors in India.

3. The current shareholding pattern is 49 % of Industry Associations and the

remaining 51% of Central and State Governments.

Which of the statements given above is/are correct?

(a) 1 and 2 only

(b) 2 only

(c) 1 and 3 only

(d) 1, 2 and 3

Solution: A

Page 33: SIMPLYFYING IAS EXAM PREPARATION - INSIGHTSIAS · Insta 75 Days Revision Plan for UPSC Civil Services Prelims – 2020 This document is the compilation of 100 questions that are part

INSTA 75 Days REVISION PLAN for Prelims 2020 - InstaTests

www.insightsonindia.com 31 Insights IAS

• Invest India, set up in 2009, is a non-profit venture under the Department for

Promotion of Industry and Internal Trade, Ministry of Commerce and Industry,

Government of India.

• We are the National Investment Promotion and Facilitation Agency of India and act

as the first point of reference for investors in India.

• As the national investment promotion and facilitation agency, Invest India focuses on

sector-specific investor targeting and development of new partnerships to enable

sustainable investments in India. In addition to a core team that focuses on sustainable

investments, Invest India also partners with substantial investment promotion

agencies and multilateral organizations. Invest India also actively works with several

Indian states to build capacity as well as bring in global best practices in investment

targeting, promotion and facilitation areas.

• Invest India was formed in 2009 under Section 25 of the Companies Act 1956 for

promotion of foreign investment with 49% equity of the then Department of Industrial

Policy and Promotion, Ministry of Commerce and Industry and 51% shareholding by

FICCI. The current shareholding pattern of Invest India is 51 % of Industry Associations

(i.e. 17% each of FICCI, CII & NASSCOM) and the remaining 49% of Central and 19 State

Governments.

• Department for Promotion of Industry and Internal Trade conducts regular review of

the performance of Invest India. Further, the Board of Directors, under chairmanship

of Secretary, Department for Promotion of Industry and Internal Trade, including

nominees from Government of India, FICCI, CII and NASSCOM manages and oversee

the overall operations, direction and strategy of the company. Regular meetings of the

Board of Directors are held to monitor the operational and overall performance of

Invest India.

32. Agreement on Reciprocal Logistics Support (ARLS), sometime seen in the news, is a

defence agreement between which of the following countries?

(a) India and United States

(b) India and Japan

(c) India and Russia

(d) India and France

Solution: C

Page 34: SIMPLYFYING IAS EXAM PREPARATION - INSIGHTSIAS · Insta 75 Days Revision Plan for UPSC Civil Services Prelims – 2020 This document is the compilation of 100 questions that are part

INSTA 75 Days REVISION PLAN for Prelims 2020 - InstaTests

www.insightsonindia.com 32 Insights IAS

Agreement on Reciprocal Logistics Support (ARLS)

• India and Russia are finalizing a defence agreement that will simplify interoperability

and enable military platforms to receive support and supplies across bases in both

nations.

• This will be beneficial for the Indian Navy, which has a large number of Russian origin

ships that will get access to Russian ports for supplies and refueling. It would be crucial

for joint exercises.

• It is an arrangement that will allow access to India and Russia, to each other’s military

facilities for supplies and fuel.

https://www.thehindu.com/news/national/india-russia-to-conclude-mutual-logistics-

agreement/article29881623.ece

33. Consider the following statements regarding Indian Ports

1. Major Ports are under the Union List while the Non-Major Ports are under the

State List of the Constitution of India.

2. Major Ports are under the administrative control of Government of India while

the Non-major ports are governed by the respective Maritime State

Governments.

3. Major ports handle more than 50% of sea-borne traffic.

Which of the statements given above is/are correct?

(a) 2 only

(b) 2 and 3 only

(c) 1 and 3 only

(d) 1, 2 and 3

Solution: B

• Indian Ports are broadly classified as Major Ports and Non-Major Ports. The Major

Ports are under the Union List while the Non-Major Ports are under the Concurrent

List of the Constitution of India. Hence, Major Ports are under the administrative

control of Government of India while the Non-major ports are governed by the

respective Maritime State Governments.

• Major Ports are defined in Section 3(8) of the Indian Ports Act 1908 to mean any port

which the Central Government may by notification in the Official Gazette declare, or

may under any law for the time being in force have declared, to be a major port.

• India has 12 major ports, which handle about 58% of sea-borne traffic. These are

Kolkata (including Dock Complex at Haldia), , Visakhapatnam, Chennai, V.O.

Page 35: SIMPLYFYING IAS EXAM PREPARATION - INSIGHTSIAS · Insta 75 Days Revision Plan for UPSC Civil Services Prelims – 2020 This document is the compilation of 100 questions that are part

INSTA 75 Days REVISION PLAN for Prelims 2020 - InstaTests

www.insightsonindia.com 33 Insights IAS

Chidambaranar (Tuticorin), Cochin, New Mangalore, Mormugao, Jawaharlal Nehru

Port Trust (JNPT), Mumbai, Kandla and Ennore. Of these, Ennore Port Ltd is a company

and the remaining 11 are Port Trusts, governed by the provisions of the Major Port

trusts Act, 1963.

• Under the Indian Ports Act, 1908, the Government has declared the Port Blair Port

with its territorial jurisdiction over all ports of Andaman & Nicobar Islands, – as a major

port w.e.f. 1 June, 2010. All major provisions of the Major Port Trusts Act, 1963 has

become applicable to the major port of Port Blair from 1 June, 2010. Though

Government has intended it to be the next major port, no further action has been

taken on the same.

• India has around 200 Non-Major ports, which handle about 42% of sea-borne traffic.

• To allow the competitive market forces to play a greater role in determination of tariff

at Major Port Trusts, the Government has issued two policy guidelines viz. Guidelines

for Determination of Tariff for Projects at Major Ports, 2013 and Guidelines for

Determination of Tariff for Major Port Trusts, 2015. These guidelines impart flexibility

to the PPP operators in the Major Ports and Major Ports owned terminals in

determining their tariff, subject to a ceiling rate.

34. Consider the following statements regarding Local Area Banks (LABs)

1. They provide services in a limited area of operation, i.e., primarily in rural and

semi-urban areas, comprising three contiguous districts.

2. LABs were required to have a minimum capital of Rs. 50 crores.

Which of the statements given above is/are correct?

(a) 1 only

(b) 2 only

(c) Both 1 and 2

(d) Neither 1 nor 2

Solution: A

• The Local Area Banks (LABs) are small private banks, conceived as low-cost structures

which would provide efficient and competitive financial intermediation services in a

limited area of operation, i.e., primarily in rural and semi-urban areas, comprising

three contiguous districts.

• LABs were set up to enable the mobilization of rural savings by local institutions and,

at the same time, to make them available for investments in the local areas.

Page 36: SIMPLYFYING IAS EXAM PREPARATION - INSIGHTSIAS · Insta 75 Days Revision Plan for UPSC Civil Services Prelims – 2020 This document is the compilation of 100 questions that are part

INSTA 75 Days REVISION PLAN for Prelims 2020 - InstaTests

www.insightsonindia.com 34 Insights IAS

• LABs were created following an announcement made by the then Finance Minister in

the Union Budget in August 1996. RBI issued guidelines for setting up of Local Area

Banks (LABs) vide its Press Release dated August 24, 1996.

• LABs were required to have a minimum capital of Rs. 5 crores.

• The promoters of the bank may comprise of private individuals, corporate entities,

trusts and societies with a minimum capital contribution of Rs. 2 crores.

• The area of operation of LAB is limited to a maximum of three geographically

contiguous districts and are allowed to open branches only in its area of operation.

• Since LABs are being set up in district towns, their activities are focused on the local

customers with lending primarily to agriculture and allied activities, small scale

industries, agro-industrial activities, trading activities and the non-farm sector. LABs

are also required to observe the priority sector lending targets at 40% of net bank

credit (NBC) as applicable to other domestic banks. Within the above target, these

banks will adhere to the requirement of lending at least 25% of their priority sector

deployments (10% of NBC) to the weaker sections.

• Around 5 LABs were licensed by 2002 under Section 22 of the Banking Regulation Act

1949 from amongst hundreds of applicants. Presently, four LABs are functioning

satisfactorily.

• In 2014, RBI has permitted LABs to be converted into small finance banks subject to

them meeting the prescribed eligibility criteria.

35. Consider the following statements regarding National Nutrition Survey

1. The survey for the first time proved the coexistence of obesity and under

nutrition.

2. Around 10% of children in the age group of 5 to 9 years and adolescents in the

age group 10 to 19 years are pre – diabetic

Which of the statements given above is/are correct?

(a) 1 only

(b) 2 only

(c) Both 1 and 2

(d) Neither 1 nor 2

Solution: C

National Nutrition Survey

• With the help of UNICEF, the Ministry of Health and Family Welfare recently

conducted the first-ever comprehensive National Nutrition Survey.

Page 37: SIMPLYFYING IAS EXAM PREPARATION - INSIGHTSIAS · Insta 75 Days Revision Plan for UPSC Civil Services Prelims – 2020 This document is the compilation of 100 questions that are part

INSTA 75 Days REVISION PLAN for Prelims 2020 - InstaTests

www.insightsonindia.com 35 Insights IAS

• The survey recorded malnutrition that included micronutrient deficiencies and details

of non-communicable diseases such as diabetes, hypertension, cholesterol and kidney

function in children and adolescents

• The survey for the first time proved the coexistence of obesity and under nutrition.

• Around 10% of children in the age group of 5 to 9 years and adolescents in the age

group 10 to 19 years are pre – diabetic. The survey for the first time proved the

coexistence of obesity and under nutrition.

• One in five children in the age group 5 to 9 years were stunted

https://www.insightsonindia.com/2019/10/01/national-nutrition-survey/

36. Which of the following is/are considered as minor minerals in India?

1. Stones used for making household utensils

2. Bentonite

3. Shale when used for building material

4. Mica

Select the correct answer using the code given below:

(a) 1, 2 and 3 only

(b) 2, 3 and 4 only

(c) 1, 2 and 4 only

(d) 1, 2, 3 and 4

Solution: D

• In India, the minerals are classified as minor minerals and major minerals.

• According to section 3(e) of the Mines and Minerals (Development and Regulation)

Act, 1957 “Minor Minerals” means building stones, gravel, ordinary clay, ordinary

sand other than sand used for prescribed purposes, and any other mineral which the

Central Government may, by notification in the Official Gazette, declare to be a minor

mineral. (For the purposes of this Act, the word “minerals” includes all minerals except

mineral oils- natural gas and petroleum)

• Major minerals are those specified in the first schedule appended in the Mines and

Minerals (Development and Regulation) Act, 1957 (MMDR Act 1957) and the

common major minerals are Lignite, Coal, Uranium, iron ore, gold etc. It may be noted

that there is no official definition for “major minerals” in the MMDR Act. Hence,

whatever is not declared as a “minor mineral” may be treated as the major mineral.

• The major-minor classification has nothing to do with the quantum /availability of

these minerals, though it is correlated with the relative value of these minerals.

Page 38: SIMPLYFYING IAS EXAM PREPARATION - INSIGHTSIAS · Insta 75 Days Revision Plan for UPSC Civil Services Prelims – 2020 This document is the compilation of 100 questions that are part

INSTA 75 Days REVISION PLAN for Prelims 2020 - InstaTests

www.insightsonindia.com 36 Insights IAS

Further, this classification is based more on their end use, rather than level of

production, level of mechanization, export and import etc. (eg. Sand can be a major

mineral or a minor mineral depending on where it is used; same is the case for

limestone.)

• India produces as many as 88 minerals which include 4 fuel minerals, 3 atomic

minerals, 26 metallic & non-metallic minerals and 55 minor minerals (including

building and other materials and the recently notified 31 additional minerals).

• The central government has the power to notify “minor minerals” under section 3 (e)

of the MMDR Act, 1957. On the other hand, as per Section 15 of the MMDR Act, 1957

State Governments have complete powers for making Rules for grant of concessions

in respect of extraction of minor minerals and levy and collection of royalty on minor

minerals.

In addition to the minor minerals specified in Section 3(e) of the MMDR Act, the Central

Government has declared the following minerals as minor minerals:

• boulder,

• shingle,

• chalcedony pebbles used for ball mill purposes only,

• lime shell, kankar and limestone used in kilns for manufacture of lime used as building

material,

• murrum,

• brick-earth,

• fuller’s earth,

• bentonite,

• road metal,

• reh-matti,

• slate and shale when used for building material,

• marble,

• stone used for making household utensils,

• quartzite and sandstone when used for purposes of building or for making road metal

and household utensils,

• saltpeter and

• ordinary earth (used or filling or leveling purposes in construction or embankments,

roads, railways, building).

Mica;

Ochre;

Pyrophyllite;

Quartz;

Quartzite;

Sand (Others);

Shale;

Page 39: SIMPLYFYING IAS EXAM PREPARATION - INSIGHTSIAS · Insta 75 Days Revision Plan for UPSC Civil Services Prelims – 2020 This document is the compilation of 100 questions that are part

INSTA 75 Days REVISION PLAN for Prelims 2020 - InstaTests

www.insightsonindia.com 37 Insights IAS

37. Consider the following statements regarding Mumbai Inter-Bank Offer Rate (MIBOR)

and Mumbai Inter-Bank Bid Rate (MIBID)

1. They are the benchmark rates at which Indian banks lend and borrow money to

each other.

2. MIBID is the rate at which banks would like to borrow from other banks and

MIBOR is the rate at which banks are willing to lend to other banks.

Which of the statements given above is/are correct?

(a) 1 only

(b) 2 only

(c) Both 1 and 2

(d) Neither 1 nor 2

Solution: C

• Mumbai Inter-Bank Offer Rate (MIBOR) and Mumbai Inter-Bank Bid Rate (MIBID)

are the benchmark rates at which Indian banks lend and borrow money to each other.

The bid is the price at which the market would buy and the offer (or ask) is the price

at which the market would sell. These rates reflect the short-term funding costs of

major banks. In other words, MIBOR reflects the price at which short term funds are

made available to participating banks.

• MIBID is the rate at which banks would like to borrow from other banks and MIBOR is

the rate at which banks are willing to lend to other banks. Contrary to general

perception, MIBID is not the rate at which banks attract deposits from other banks.

• MIBOR is the Indian version of London Interbank Offer Rate (LIBOR). MIBOR is fixed

for overnight to 3 month long funds and these rates are published every day at a

designated time. Of the above tenors, the overnight MIBOR is the most widely used

one which is used for pricing and settlement of Overnight Index Swaps (OIS).

Corporates use the OIS for hedging their interest rate risks. The MIBID/MIBOR rate is

also used as a bench mark rate for majority of deals struck for Interest Rate Swaps

(IRS), Forward Rate Agreements (FRA), Floating Rate Debentures and Term Deposits.

The aggregate amount of outstanding interbank/Primary Dealers (PD) notional

principal referenced to MIBOR remained at INR 16,847.6 billion as on October 31,

2013

Financial Benchmarks

• MIBOR, MIBID etc. are all financial benchmarks.

Page 40: SIMPLYFYING IAS EXAM PREPARATION - INSIGHTSIAS · Insta 75 Days Revision Plan for UPSC Civil Services Prelims – 2020 This document is the compilation of 100 questions that are part

INSTA 75 Days REVISION PLAN for Prelims 2020 - InstaTests

www.insightsonindia.com 38 Insights IAS

38. Consider the following statements regarding School Education Quality Index (SEQI)

1. Kerala has emerged on top among 20 large states in terms of quality of school

education.

2. It aims to bring an ‘outcomes’ focus to education policy by providing States and

UTs with a platform to identify their strengths and weaknesses

3. All seven union territories have shown significant decline in their overall

performance scores

Which of the statements given above is/are correct?

(a) 1 only

(b) 1 and 3 only

(c) 1 and 2 only

(d) 2 and 3 only

Solution: C

School Education Quality Index (SEQI): By NITI Aayog

• The School Education Quality Index (SEQI) was developed to evaluate the

performance of States and Union Territories (UTs) in the school education sector. The

index aims to bring an outcomes focus to education policy by providing States and UTs

with a platform to identify their strengths and weaknesses and undertake requisite

course corrections or policy interventions. In line with NITI Aayog’s mandate to foster

the spirit of competitive and cooperative federalism, the index strives to facilitate the

sharing of knowledge and best practices across States and UTs.

• It aims to bring an ‘outcomes’ focus to education policy by providing States and UTs

with a platform to identify their strengths and weaknesses and undertake requisite

course corrections or policy interventions.

• The index is developed through a collaborative process, including key stakeholders

such as Ministry of Human Resource and Development (MHRD), the World Bank and

sector experts.

• It consists of 30 critical indicators that assess the delivery of quality education.

• Kerala has emerged on top among 20 large states in terms of quality of school

education, followed by Rajasthan and Karnataka, [while the most-populous Uttar

Pradesh was ranked at the bottom position during 2016-17]

• The overall performance has declined in Karnataka and Uttarakhand.

• All seven union territories have shown an improvement in their overall performance

scores.

https://niti.gov.in/content/school-education-quality-index

Page 41: SIMPLYFYING IAS EXAM PREPARATION - INSIGHTSIAS · Insta 75 Days Revision Plan for UPSC Civil Services Prelims – 2020 This document is the compilation of 100 questions that are part

INSTA 75 Days REVISION PLAN for Prelims 2020 - InstaTests

www.insightsonindia.com 39 Insights IAS

39. Consider the following statements regarding Commodities transaction tax (CTT)

1. Commodities transaction tax (CTT) is a tax similar to Securities Transaction Tax

(STT), levied in India, on transactions done on the domestic commodity

derivatives exchanges.

2. The concept of CTT was first introduced in the Union Budget 2018-19.

3. All agricultural commodities are exempted from CTT.

Which of the statements given above is/are correct?

(a) 1 only

(b) 2 and 3 only

(c) 1 and 3 only

(d) 1, 2 and 3

Solution: A

• Commodities transaction tax (CTT) is a tax similar to Securities Transaction Tax (STT),

levied in India, on transactions done on the domestic commodity derivatives

exchanges. In, Finance Act, 2016 it was stipulated that transactions carried out in a

recognized commodity exchange located in an International Financial Center, where

the payments are carried out in terms of foreign currency, would be exempt from the

payment of CTT.

• Globally, commodity derivatives are also considered as financial contracts. Hence CTT

can also be considered as a type of financial transaction tax.

• The concept of CTT was first introduced in the Union Budget 2008-09 (para 179 of the

Budget Speech). The Government had then proposed to impose a commodities

transaction tax (CTT) of 0.017% (equivalent to the rate of equity futures at that point

of time). However, it was withdrawn subsequently as the market was nascent then

and any imposition of transaction tax might have adversely affected the growth of

organized commodities derivatives markets in India. This has helped Indian

commodity exchanges to grow to global standards (MCX is the world’s number 3

commodity exchange; Globally, MCX is No. 1 in Gold and Silver, No. 2 in Natural gas

and No. 3 in Crude Oil)

• In the Union Budget 2013-14 (para 149 of the Budget Speech) CTT has been re-

introduced, however, only for non-agricultural commodity futures at the rate of 0.01%

(which is equivalent to the rate of equity futures). Along with this, transactions in

commodity derivatives have been declared to be made non-speculative; and hence

for traders in the commodity derivative segment, any losses arising from such

transactions can be set off against income from any other source (similar provisions

are applicable for the securities market transactions).

• A separate provision for CTT has been made in the Finance Act, 2013 (Chapter VII).

Page 42: SIMPLYFYING IAS EXAM PREPARATION - INSIGHTSIAS · Insta 75 Days Revision Plan for UPSC Civil Services Prelims – 2020 This document is the compilation of 100 questions that are part

INSTA 75 Days REVISION PLAN for Prelims 2020 - InstaTests

www.insightsonindia.com 40 Insights IAS

• The CTT rules were notified by Department of Revenue, Ministry of Finance on 19

June 2013 (Notification No. 46 of 2013; S.O. 1769 (E)), with effect from 1 July 2013. As

per the notification, only 23 agricultural commodities were exempted from CTT.

However, in February 2015 a revised list of 61 commodities was notified including

certain commodities where trading is currently not taking place. Like STT, the

commodity exchanges have been entrusted to collect CTT on behalf of Government

of India.

• Like all financial transaction taxes, CTT aims at discouraging excessive speculation,

which is detrimental to the market and to bring parity between securities market and

commodities market such that there is no tax / regulatory arbitrage.

40. Consider the following statements regarding Gini index or Gini coefficient

1. It is often used as a gauge of economic inequality, measuring income distribution

or, less commonly, wealth distribution among a population.

2. The coefficient ranges from 0 (or 0%) to 1 (or 100%), with 0 representing perfect

equality and 1 representing perfect inequality.

3. The Gini index is often represented graphically through the Lorenz curve.

Which of the statements given above is/are correct?

(a) 1 and 2 only

(b) 2 and 3 only

(c) 1 and 3 only

(d) 1, 2 and 3

Solution: D

• The Gini index or Gini coefficient is a statistical measure of distribution developed by

the Italian statistician Corrado Gini in 1912. It is often used as a gauge of economic

inequality, measuring income distribution or, less commonly, wealth distribution

among a population. The coefficient ranges from 0 (or 0%) to 1 (or 100%), with 0

representing perfect equality and 1 representing perfect inequality. Values over 1 are

theoretically possible due to negative income or wealth.

• The Gini index is a simple measure of the distribution of income across income

percentiles in a population.

• A higher Gini index indicates greater inequality, with high income individuals receiving

much larger percentages of the total income of the population.

• Global inequality as measured by the Gini index increased over the 19th and 20th

centuries, but has declined in more recent years.

Page 43: SIMPLYFYING IAS EXAM PREPARATION - INSIGHTSIAS · Insta 75 Days Revision Plan for UPSC Civil Services Prelims – 2020 This document is the compilation of 100 questions that are part

INSTA 75 Days REVISION PLAN for Prelims 2020 - InstaTests

www.insightsonindia.com 41 Insights IAS

• Because of data and other limitations, the Gini index may overstate income inequality

and can obscure important information about income distribution.

• The Gini index is often represented graphically through the Lorenz curve, which shows

income (or wealth) distribution by plotting the population percentile by income on the

horizontal axis and cumulative income on the vertical axis. The Gini coefficient is equal

to the area below the line of perfect equality (0.5 by definition) minus the area below

the Lorenz curve, divided by the area below the line of perfect equality. In other

words, it is double the area between the Lorenz curve and the line of perfect equality.

41. Consider the following statements regarding Rashtriya Aavishkar Abhiyan (RAA)

1. It has been launched under Atal Innovation Mission

2. It will target students in the age group of 6 – 18 years.

3. It aims to encourage children towards learning Science and Mathematics.

Which of the statements given above is/are correct?

(a) 1 only

(b) 2 and 3 only

(c) 1 and 3 only

(d) 1, 2 and 3

Solution: B

Rashtriya Aavishkar Abhiyan (RAA):

• It was launched in 2015.

• Ministry of Human Resource Development has launched the Rashtriya Avishkar

Abhiyan (RAA).

• It is a convergent framework across School Education and Higher Education to

motivate children of the age group from 6-18 years in learning Science, Mathematics

and Technology through observation, experimentation, inference drawing, model

building, etc. both through inside and outside classroom activities and processes. It

seeks to create curiosity, excitement and spirit of innovation and exploration amongst

school children by encouraging higher education institutions to become Mentoring

Institutions and assist secondary and elementary schools in the study of Science and

Mathematics.

• Major interventions under RAA provided under Integrated scheme for School

Education – Samagra Shiksha, include conduct of Science Exhibition, Book Fair, Quiz

Competition, exposure/study visits for students to Higher Education institutions,

participation of students in Inter-school/State/National level Science & Maths

Page 44: SIMPLYFYING IAS EXAM PREPARATION - INSIGHTSIAS · Insta 75 Days Revision Plan for UPSC Civil Services Prelims – 2020 This document is the compilation of 100 questions that are part

INSTA 75 Days REVISION PLAN for Prelims 2020 - InstaTests

www.insightsonindia.com 42 Insights IAS

Competitions/Olympiads, strengthening of School Science and Mathematics

laboratories, use of teaching-learning equipment and material including Digital

models and use of technology in Science and Mathematics teaching.

42. Consider the following statements regarding National Bank for Agriculture and Rural

Development (NABARD)

1. NABARD today is fully owned by RBI.

2. It was setup based on the recommendation of B. Sivaraman Committee.

Which of the statements given above is/are correct?

(a) 1 only

(b) 2 only

(c) Both 1 and 2

(d) Neither 1 nor 2

Solution: B

National Bank for Agriculture and Rural Development (NABARD)

• The importance of institutional credit in boosting rural economy has been clear to the

Government of India right from its early stages of planning. Therefore, the Reserve

Bank of India (RBI) at the insistence of the Government of India, constituted a

Committee to Review the Arrangements For Institutional Credit for Agriculture and

Rural Development (CRAFICARD) to look into these very critical aspects. The

Committee was formed on 30 March 1979, under the Chairmanship of Shri B.

Sivaraman, former member of Planning Commission, Government of India.

• The Committee’s interim report, submitted on 28 November 1979, outlined the need

for a new organizational device for providing undivided attention, forceful direction

and pointed focus to credit related issues linked with rural development. Its

recommendation was formation of a unique development financial institution which

would address these aspirations and formation of National Bank for Agriculture and

Rural Development (NABARD) was approved by the Parliament through Act 61 of

1981.

• NABARD came into existence on 12 July 1982 by transferring the agricultural credit

functions of RBI and refinance functions of the then Agricultural Refinance and

Development Corporation (ARDC). It was dedicated to the service of the nation by

the late Prime Minister Smt. Indira Gandhi on 05 November 1982. Set up with an initial

capital of Rs.100 crore, its’ paid up capital stood at Rs.12,580 crore as on 31 March

2019. Consequent to the revision in the composition of share capital between

Government of India and RBI, NABARD today is fully owned by Government of India.

Page 45: SIMPLYFYING IAS EXAM PREPARATION - INSIGHTSIAS · Insta 75 Days Revision Plan for UPSC Civil Services Prelims – 2020 This document is the compilation of 100 questions that are part

INSTA 75 Days REVISION PLAN for Prelims 2020 - InstaTests

www.insightsonindia.com 43 Insights IAS

43. Consider the following statements regarding CriSidEx

1. It was launched by CRISIL and SIDBI.

2. It is a composite index which measures business sentiment of Micro and Small

Enterprises.

3. A scale of 0 (extremely negative) to 100 (extremely positive).

Which of the statements given above is/are correct?

(a) 1 and 2 only

(b) 2 and 3 only

(c) 1 and 3 only

(d) 1, 2 and 3

Solution: A

About CriSidEx:

CRISIL-SIDBI MSE Sentiment Index, or CriSidEx

• It was launched by CRISIL and SIDBI.

• CriSidEx is a composite index based on a diffusion index of 8 parameters and measures

MSE business sentiment on a scale of 0 (extremely negative) to 200 (extremely

positive). CriSidEx will have 2 indices, one for the ‘survey quarter’ and another for the

‘next quarter’ once a trend emerges after few rounds of the survey, providing

independent time series data.

• Benefits: The crucial benefit of CriSidEx is that its readings will flag potential

headwinds and changes in production cycles and thus help improve market

efficiencies. And by capturing the sentiment of exporters and importers, it will also

offer actionable indicators on foreign trade.

Significance of MSME:

• MSME sector is backbone of the economy. It is one of the largest employer in the

country and with the vast population where employment either in government or in

the large industry itself has limited potential. This is one sector where people not only

exhibit their entrepreneurial skills, become part of large value chains but also become

job creators in the process. And that is the reason why a bulk of the jobs in

manufacturing, trading have been created in this particular sector.

Page 46: SIMPLYFYING IAS EXAM PREPARATION - INSIGHTSIAS · Insta 75 Days Revision Plan for UPSC Civil Services Prelims – 2020 This document is the compilation of 100 questions that are part

INSTA 75 Days REVISION PLAN for Prelims 2020 - InstaTests

www.insightsonindia.com 44 Insights IAS

44. Consider the following statements regarding Leader of Opposition

1. It is a constitutional post

2. He is leader of the largest party that has not less than one-tenth of the total

strength of the house

3. He plays a crucial role in appointments of CVC, CBI, CIC, Lokpal.

Which of the statements given above is/are correct?

(a) 2 and 3 only

(b) 3 only

(c) 1 and 2 only

(d) 1, 2 and 3

Solution: A

Leader of Opposition:

• The LOP is leader of the largest party that has not less than one-tenth of the total

strength of the house.

• It is a statutory post defined in the Salaries and Allowances of Leaders of Opposition

in Parliament Act, 1977.

Significance of the office:

• LoP is referred to as the ‘shadow Prime Minister’.

• She/he is expected to be ready to take over if the government falls.

• The LoP also plays an important role in bringing cohesiveness and effectiveness to the

opposition’s functioning in policy and legislative work.

• LoP plays a crucial role in bringing bipartisanship and neutrality to the appointments

in institutions of accountability and transparency – CVC, CBI, CIC, Lokpal etc.

45. Consider the following statements regarding Business Correspondent

1. Business correspondents are bank representatives and help villagers to open bank

accounts.

2. Only retired bank employees can become Business correspondents.

3. BC’s must be a permanent resident of the area in which they propose to operate.

Which of the statements given above is/are correct?

(a) 1 and 2 only

(b) 2 and 3 only

Page 47: SIMPLYFYING IAS EXAM PREPARATION - INSIGHTSIAS · Insta 75 Days Revision Plan for UPSC Civil Services Prelims – 2020 This document is the compilation of 100 questions that are part

INSTA 75 Days REVISION PLAN for Prelims 2020 - InstaTests

www.insightsonindia.com 45 Insights IAS

(c) 1 and 3 only

(d) 1, 2 and 3

Solution: C

Business Correspondent (BC)

• Business correspondents are bank representatives. They help villagers to open bank

accounts. Business Correspondents get commission from bank for every new account

opening, every transaction made via them, every loan-application processed etc. The

Business Correspondent carries a mobile device and helps villagers in banking

transactions. (Deposit money, take money out of savings account, loans etc.). The

villager gives his thumb impression or electronic signature, and gets the money.

Eligibility to become a Business Correspondent

As per the RBI guidelines, the following entities are eligible for appointment of Business

Correspondents (BCS) for banks:

• NGOs/MFls set up under Societies/Trust Acts

• Societies registered under Mutually Aided Cooperative Societies Acts or the

Cooperative Societies Acts of States

• Section 25 companies that are stand alone entities or in which NBFCs, banks, telecom

companies and other corporate entities or their holding companies did not have

equity holdings in excess of 10 per cent

• Post offices

• Retired bank employees,

• Ex-servicemen

• Retired government employees.

• Individual kirana / medical / fair price shop owners Individual Public Call Office (PCO)

operators

• Agents of Small Savings Schemes of Government of India/Insurance Companies

Individuals who own petrol pumps

• Retired teachers

• Authorized functionaries of well run Self Help Groups (SHGs) linked to banks Non

deposit taking NBFCs (non-banking finance companies) in the nature of loan

companies whose micro finance portfolio is not less than 80 per cent of their loan

outstanding in the financially excluded districts as identified by the Committee on

Financial Inclusion.

RBI has now permitted banks to engage any individual, including those operating Common

Service Centres (CSCs) as BC, subject to banks’ comfort level and their carrying out suitable

Page 48: SIMPLYFYING IAS EXAM PREPARATION - INSIGHTSIAS · Insta 75 Days Revision Plan for UPSC Civil Services Prelims – 2020 This document is the compilation of 100 questions that are part

INSTA 75 Days REVISION PLAN for Prelims 2020 - InstaTests

www.insightsonindia.com 46 Insights IAS

due diligence as also instituting additional safeguards as may be considered appropriate to

minimise the agency risks.

Appointment of BCs

• Must be a permanent resident of the area in which they propose to operate. They

should be well established, enjoy good reputation and have the confidence of the local

people.

• The ability of BCs to invest in POS machines and other equipment. In case of individuals

Selected as BCS, the criterion is as under: A minimum education qualification of Xth

pass. Field Investigation /RCU for verification of residence and dealings, etc. to be

conducted.

• Credibility check – A/c with any other bank. Should open account with Bank (base

branch)

• Suitable amount of Security deposit /Bank guarantee based on business Volumes.

46. Consider the following statements regarding International Court of Justice

1. All members of the UN are parties to the statute of the ICJ.

2. Any individuals, non-governmental organizations, corporations or any other

private entities can appear before the ICJ.

3. ICJ only has jurisdiction over any dispute, if the both concerned parties submit to

the jurisdiction of ICJ.

Which of the statements given above is/are correct?

(a) 1 and 2 only

(b) 2 and 3 only

(c) 1 and 3 only

(d) 1, 2 and 3

Solution: C

International Court of Justice:

• The International Court of Justice (ICJ) is the principal judicial body of the UN.

Established in 1946 to replace the Permanent Court of International Justice, the ICJ

mainly operates under the statute of its predecessor, which is included in the UN

Charter.

• It has two primary functions: to settle legal disputes submitted by States in accordance

with established international laws, and to act as an advisory board on issues

submitted to it by authorized international organizations.

Page 49: SIMPLYFYING IAS EXAM PREPARATION - INSIGHTSIAS · Insta 75 Days Revision Plan for UPSC Civil Services Prelims – 2020 This document is the compilation of 100 questions that are part

INSTA 75 Days REVISION PLAN for Prelims 2020 - InstaTests

www.insightsonindia.com 47 Insights IAS

• Only countries are eligible to appear before the ICJ, not individuals, non-governmental

organisations, corporations or any other private entities.

• ICJ only has jurisdiction over any dispute, if the both concerned parties (states) submit

to the jurisdiction of ICJ

Members of the Court:

• The International Court of Justice is composed of 15 judges elected to nine-year terms

of office by the United Nations General Assembly and the Security Council. These

organs vote simultaneously but separately. In order to be elected, a candidate must

receive an absolute majority of the votes in both bodies. In order to ensure a measure

of continuity, one third of the Court is elected every three years. Judges are eligible

for re-election.

• All members of the UN are parties to the statute of the ICJ, and non-members may

also become parties.

Who nominates the candidates?

• Every state government, party to the Charter, designates a group who propose

candidates for the office of ICJ judges. This group includes four members/jurists of the

Permanent Court of Arbitration (machinery which enables arbitral tribunals to be set

up as desired and facilitates their work) also picked by the State. Countries not part of

the statute follow the same procedure where a group nominates the candidates.

• Each group is limited to nominate four candidates, two of whom could be of their

nationality. Within a fixed duration set by the Secretary-General, the names of the

candidates have to be sent to him/her.

What are the qualifications of ICJ judges?

• A judge should have a high moral character.

• A judge should fit to the qualifications of appointment of highest judicial officers as

prescribed by their respective states or.

• A judge should be a juri-consult of recognized competence in international law.

The 15 judges of the Court are distributed as per the regions:

• Three from Africa.

• Two from Latin America and Caribbean.

• Three from Asia.

• Five from Western Europe and other states.

• Two from Eastern Europe.

Independence of the Judges:

• Once elected, a Member of the Court is a delegate neither of the government of his

own country nor of that of any other State. Unlike most other organs of international

organizations, the Court is not composed of representatives of governments.

Members of the Court are independent judges whose first task, before taking up their

Page 50: SIMPLYFYING IAS EXAM PREPARATION - INSIGHTSIAS · Insta 75 Days Revision Plan for UPSC Civil Services Prelims – 2020 This document is the compilation of 100 questions that are part

INSTA 75 Days REVISION PLAN for Prelims 2020 - InstaTests

www.insightsonindia.com 48 Insights IAS

duties, is to make a solemn declaration in open court that they will exercise their

powers impartially and conscientiously.

• In order to guarantee his or her independence, no Member of the Court can be

dismissed unless, in the unanimous opinion of the other Members, he/she no longer

fulfils the required conditions. This has in fact never happened.

47. Which of following are major economy traits of Recovery cycle in the business cycle?

1. An upturn in aggregate (total) demand which has to be accompanied by increase

in the level of production.

2. As demand goes upward, inflation also moves upward making borrowing cheaper

for investors.

3. Production process expands and new investments become attractive.

Select the correct answer using the code given below:

(a) 1 and 2 only

(b) 2 and 3 only

(c) 1 and 3 only

(d) 1, 2 and 3

Solution: D

Recovery

An economy tries to come out of the low production phase to survive. The low production

phase might be depression, recession or slowdown with the former being the worst and rare,

government stake many new fiscal and monetary measures to boost demand and production

and ultimately a recovery in an economy is managed. The business cycle of recovery may

show the following major economy traits:

(i) an upturn in aggregate (total) demand which has to be accompanied by increase in

the level of production;

(ii) production process expands and new investments become attractive;

(iii) as demand goes upward, inflation also moves upward making borrowing cheaper for

investors;

(iv) with an upturn in production, new employment avenues are created and

unemployment rate starts declining; etc.

Page 51: SIMPLYFYING IAS EXAM PREPARATION - INSIGHTSIAS · Insta 75 Days Revision Plan for UPSC Civil Services Prelims – 2020 This document is the compilation of 100 questions that are part

INSTA 75 Days REVISION PLAN for Prelims 2020 - InstaTests

www.insightsonindia.com 49 Insights IAS

48. The port integrity campaign has been launched by

(a) International Maritime Organization

(b) Maritime safety organizations

(c) Maritime trade unions

(d) Maritime Anti-Corruption Network

Solution: D

Port integrity campaign:

• Maritime Anti-Corruption Network starts port integrity campaign in India.

• The campaign, which aims to reduce and (in the long term) eliminate integrity issues

and bottlenecks to trade during operations in Indian ports, is a collective action of

MACN, the Government of India, international organizations, and local industry

stakeholders.

• The main activities of the campaign include implementation of integrity training for

port officials and the establishment of clear escalation and reporting processes.

About MACN:

• Maritime Anti-Corruption Network (MACN) is a global business network of over 110

companies working to tackle corruption in the maritime industry.

• Established in 2011 by a small group of committed maritime companies.

49. Consider the following statements regarding Minimum Support Price (MSP) for Minor

Forest Produce (MFP)

1. MSP rate are decided by Ministry of Tribal Affairs.

2. The Scheme is applicable only in eight States having Schedule areas as listed in the

Fifth Schedule.

3. It is a Centrally Sponsored Scheme.

Which of the statements given above is/are correct?

(a) 1 only

(b) 2 and 3 only

(c) 1 and 3 only

(d) 1, 2 and 3

Page 52: SIMPLYFYING IAS EXAM PREPARATION - INSIGHTSIAS · Insta 75 Days Revision Plan for UPSC Civil Services Prelims – 2020 This document is the compilation of 100 questions that are part

INSTA 75 Days REVISION PLAN for Prelims 2020 - InstaTests

www.insightsonindia.com 50 Insights IAS

Solution: C

• Under the scheme “Mechanism for Marketing of Minor Forest Produce through

Minimum Support Price and development of Value chain for MFP” Minimum Support

Price (MSP) for Minor Forest Produce (MFP) has been fixed for select MFP. The scheme

is designed as a social safety net for improvement of livelihood of MFP gatherers by

providing them fair price for the MFPs they collect.

Coverage

• The Scheme has been implemented in eight States having Schedule areas as listed in

the Fifth Schedule of the constitution of India. From November 2016, the scheme is

applicable in all States.

What is this scheme all about?

• The Union Cabinet, in 2013, approved a Centrally Sponsored Scheme for marketing of

non-nationalized / non monopolized Minor Forest Produce (MFP) and development

of a value chain for MFP through Minimum Support Price (MSP).

• This was a measure towards social safety for MFP gatherers, who are primarily

members of the Scheduled Tribes (STs) most of them in Left Wing Extremism (LWE)

areas.

• The scheme had Rs. 967.28 crore as Central Government share and Rs. 249.50 crore

as the States share for the current Plan period.

• Key features of the scheme: Ensure that the tribal population gets a remunerative

price for the produce they collect from the forest and provide alternative employment

avenues to them.

• Establish a system to ensure fair monetary returns for forest dweller’s efforts in

collection, primary processing, storage, packaging, transportation etc, while ensuring

sustainability of the resource base.

• Get them a share of revenue from the sales proceeds with costs deducted.

Coverage:

• Earlier, the scheme was extended only to Scheduled Areas in eight states and fixed

MSPs for 12 MFPs. Later expanded to all states and UTs. Total number of MFPs

covered under the list include more than 40 items.

• Implementation: The responsibility of purchasing MFP on MSP will be with State

designated agencies.

• To ascertain market price, services of market correspondents would be availed by the

designated agencies particularly for major markets trading in MFP.

• The scheme supports primary value addition as well as provides for supply chain

infrastructure like cold storage, warehouses etc.

Page 53: SIMPLYFYING IAS EXAM PREPARATION - INSIGHTSIAS · Insta 75 Days Revision Plan for UPSC Civil Services Prelims – 2020 This document is the compilation of 100 questions that are part

INSTA 75 Days REVISION PLAN for Prelims 2020 - InstaTests

www.insightsonindia.com 51 Insights IAS

• The Ministry of Tribal Affairs will be the nodal Ministry for implementation and

monitoring of the scheme. The Minimum Support Price would be determined by the

Ministry with technical help of TRIFED.

• States have been given freedom for fixing MSP 10% above or below the MSP rate

decided by Ministry of Tribal Affairs.

Significance of the scheme:

• The Minor Forest Produce (MFP), also known as Non-Timber Forest Produce (NTFP),

is a major source of livelihood and provides essential food, nutrition, medicinal needs

and cash income to a large number of STs who live in and around forests. An estimated

100 million forest dwellers depend on the Minor Forest Produce for food, shelter,

medicines, cash income, etc.

• However, MFP production is highly dispersed spatially because of the poor

accessibility of these areas and competitive market not having evolved. Consequently,

MFP gatherers who are mostly poor are unable to bargain for fair prices. This package

of intervention can help in organizing unstructured MFP markets.

50. Consider the following statements regarding Urban Haats

1. It has been launched by Ministry of Culture

2. It aims to provide direct marketing facilities to the craft persons and weavers.

Which of the statements given above is/are correct?

(a) 1 only

(b) 2 only

(c) Both 1 and 2

(d) Neither 1 nor 2

Solution: B

Urban Haats:

• It is an initiative of the Ministry of Textiles of the Government of India.

• The objective of the scheme “Infrastructure and Technology Support” is to setup a

permanent marketing infrastructure in big towns/ metropolitan cities to provide

direct marketing facilities to the handicrafts artisans/handloom weavers.

• The scheme is implemented through State Handicrafts/Handlooms Development

Corporations/Tourism Development Corporations/ Urban Local Bodies with sufficient

financial resources and organizational capacity to implement the project.

Page 54: SIMPLYFYING IAS EXAM PREPARATION - INSIGHTSIAS · Insta 75 Days Revision Plan for UPSC Civil Services Prelims – 2020 This document is the compilation of 100 questions that are part

INSTA 75 Days REVISION PLAN for Prelims 2020 - InstaTests

www.insightsonindia.com 52 Insights IAS

• The financial ceiling for Urban Haat is Rs. 300 lakh for each unit. 80% of the admissible

amount is borne by the Office of the Development Commissioner (Handicrafts) and

20% contributed by the implementing agency.

DAY – 67 (InstaTest-67)

51. Consider the following statements regarding Annual study of state-level budgets

released by RBI

1. Except during 2016-17, state governments have regularly met their fiscal deficit

target of 3% of GDP

2. States now have a greater role to play in determining India’s GDP than the Centre.

3. Most states ended up meeting the fiscal deficit target not by increasing their

revenues but by reducing their expenditure

Which of the statements given above is/are correct?

(a) 1, 2 and 3

(b) 2 only

(c) 1 and 2 only

(d) 3 only

Solution: A

Annual study of state-level budgets is released by RBI.

Important Observations:

• Except during 2016-17, state governments have regularly met their fiscal deficit

target of 3% of GDP

• However, most states ended up meeting the fiscal deficit target not by increasing their

revenues but by reducing their expenditure and increasingly borrowing from the

market

• States now have a greater role to play in determining India’s GDP than the Centre-

States now spend one-and-a-half times more than the Union government

• If a state, or all the states in aggregate, find it difficult to raise revenues, a rising

mountain of debt — captured in the debt-to-GDP ratio — could start a vicious cycle.

Page 55: SIMPLYFYING IAS EXAM PREPARATION - INSIGHTSIAS · Insta 75 Days Revision Plan for UPSC Civil Services Prelims – 2020 This document is the compilation of 100 questions that are part

INSTA 75 Days REVISION PLAN for Prelims 2020 - InstaTests

www.insightsonindia.com 53 Insights IAS

Extra Reading

The Fiscal Responsibility and Budget Management (FRBM) Act

• The Act was enacted in 2003 which set targets for the government to reduce fiscal

deficits. The targets were put off several times.

• Hence, in May 2016, the government set up a committee under NK Singh to review

the FRBM Act.

• The committee recommended that the government should target a fiscal deficit of 3%

of the GDP in the years up to March 31, 2020, cut it to 2.8% in 2020-21 and to 2.5% by

2023.

https://www.rbi.org.in/Scripts/AnnualPublications.aspx?head=State%20Finances%20:%20A

%20Study%20of%20Budgets

https://indianexpress.com/article/explained/rbi-state-budget-explained-gdp-economy-

6056708/

52. Consider the following statements regarding entrepreneurial intensity in India

1. A 10 per cent increase in registration of new firms per district-year yields an 18

per cent increase in GDDP (Gross Domestic District Product).

2. The peninsular states dominate entry of new firms and is restricted just to a few

metropolitan cities.

Which of the statements given above is/are correct?

(a) 1 only

(b) 2 only

(c) Both 1 and 2

(d) Neither 1 nor 2

Solution: D

• Comparative entrepreneurial intensity: Between 2006 and 2016, the mean (median)

number of new firms registered per year per 1000 workers was 0.10(0.11). In contrast,

the mean (median) entrepreneurial intensity for the United Kingdom and the United

States was 12.22 (11.84) and 12.12 (11.81) respectively.

Dominance in Informal sector:

• India has low rates of entrepreneurship in the formal economy. In contrast to the

other countries, a large number of India’s enterprises operate in the informal

economy which is not captured in these data.

Page 56: SIMPLYFYING IAS EXAM PREPARATION - INSIGHTSIAS · Insta 75 Days Revision Plan for UPSC Civil Services Prelims – 2020 This document is the compilation of 100 questions that are part

INSTA 75 Days REVISION PLAN for Prelims 2020 - InstaTests

www.insightsonindia.com 54 Insights IAS

• Entrepreneurship and GDP: The entrepreneurial activity is related to economic

growth as it acts as an engine of economic growth and change in India. Regarding this

survey observes the following:

o A 10 per cent increase in registration of new firms per district-year yields a 1.8per

cent increase in GDDP (Gross Domestic District Product)

o Though the peninsular states dominate entry of new firms, entrepreneurship is

dispersed across India and is not restricted just to a few metropolitan cities.

o Spatial Heterogeneity exits in distribution of Entrepreneurial Activity in India i.e.

varied level of entrepreneurial activity across the districts. The survey finds that all

four regions except certain eastern states in India demonstrate strong growth in

entrepreneurial activity over time.

o While Gujarat’s labour reforms are viewed as pro-worker, the state has also

passed other regulations that improve ease of doing business, including reduction

in compliance burden, transparent and timely processing of approval and renewal

of applications, etc.

o Rajasthan too has introduced several reforms that are viewed as pro-employer.

For example, to reduce the influence of trade unions, the state has increased the

costs of union formation by increasing the minimum membership requirement to

form a union to 30 per cent of the total workforce at an establishment, up from

15 per cent earlier.

53. Consider the following statements regarding SENSEX growth and pro- business

sentiments

1. The share of services sector in the total number of companies on the Sensex has

changed from being negligible in the 1980s to the dominant status today.

2. The difference between the sizes of the largest and smallest firms is rapidly

increasing.

Which of the statements given above is/are correct?

(a) 1 only

(b) 2 only

(c) Both 1 and 2

(d) Neither 1 nor 2

Solution: A

Page 57: SIMPLYFYING IAS EXAM PREPARATION - INSIGHTSIAS · Insta 75 Days Revision Plan for UPSC Civil Services Prelims – 2020 This document is the compilation of 100 questions that are part

INSTA 75 Days REVISION PLAN for Prelims 2020 - InstaTests

www.insightsonindia.com 55 Insights IAS

Pro-Business, Creative destruction and Wealth creation

Other than the SENSEX growth, pro- business sentiments also led to the following:

o The years following 1991 liberalization saw the rapid emergence of new firms, new

ideas, new technologies and new operating processes, causing a steep revision of the

Sensex in 1996.

o New sectors like banks and financials entered the index for the first time, eroding the

predominance of the manufacturing sector on the index, placing the services sector

on the map for the first time.

o The diversity of sectors in the Sensex steadily increased over time following market

reforms. The initial Sensex of 1986 was dominated by the Materials and Consumer

Discretionary sectors, accounting for two-thirds of the firms on the Sensex. Sectors

like financials, telecommunications and information technology were non-existent in

the index then. With the entry of these new sectors, today’s Sensex is far less

concentrated than the Sensex of the 1980sand 1990s, and mirrors the far lower

sectoral concentration of the Indian economy as a whole.

o Over the years, the share of services sector in the total number of companies on the

Sensex has changed from being negligible in the 1980s to the dominant status today.

The rising share in market capitalization has been accomplished by a rise in number of

companies rather than a rise in size of incumbents, suggesting greater competition

within these sectors.

o The difference between the sizes of the largest and smallest firms is rapidly shrinking,

and consequently monopoly power in the economy is declining and making way for

more competitive markets. In 1991, the size of the largest firm was nearly 100 times

the smallest firm which got reduced to 12 in 2018.

54. Consider the following statements regarding Information Fusion Centre (IFC) for the

Indian Ocean Region (IOR)

1. It was established to gather intelligence on the movement of enemy submarines

2. It also facilitates the exchange of information on “white shipping” with countries

in the region to improve maritime domain awareness in the Indian Ocean.

Which of the statements given above is/are correct?

(a) 1 only

(b) 2 only

(c) Both 1 and 2

(d) Neither 1 nor 2

Solution: B

Page 58: SIMPLYFYING IAS EXAM PREPARATION - INSIGHTSIAS · Insta 75 Days Revision Plan for UPSC Civil Services Prelims – 2020 This document is the compilation of 100 questions that are part

INSTA 75 Days REVISION PLAN for Prelims 2020 - InstaTests

www.insightsonindia.com 56 Insights IAS

About IFC- IOR:

• The IFC-IOR was established with the vision of strengthening maritime security in the

region and beyond, by building a common coherent maritime situation picture and

acting as a maritime information hub for the region.

• The IFC has been established at the Navy’s Information Management and Analysis

Centre (IMAC) in Gurugram, which is th e single point centre linking all the coastal

radar chains to generate a seamless real-time picture of the nearly 7,500-km coastline.

• Through this Centre, information on “white shipping”, or commercial shipping, will

be exchanged with countries in the region to improve maritime domain awareness in

the Indian Ocean.

https://www.insightsonindia.com/2019/10/07/information-fusion-centre-ifc-for-the-indian-

ocean-region-ior-4/

55. Consider the following statements regarding Economic freedom

1. The Index of Economic Freedom is brought out by the Heritage Foundation.

2. The Global Economic Freedom Index is brought out by the Fraser Institute.

Which of the statements given above is/are correct?

(a) 1 only

(b) 2 only

(c) Both 1 and 2

(d) Neither 1 nor 2

Solution: C

Economic freedom enhances wealth creation by enabling efficient allocation of

entrepreneurial resources and energy to productive activities, thereby promoting economic

dynamism.

Facts:

o The Index of Economic Freedom, which is brought out by the Heritage Foundation,

India was categorized as ‘mostly unfree’ with a score of 55.2 in2019 ranking the

Indian economy 129th among 186 countries, i.e., in the bottom 30 per cent of

countries.

o The Global Economic Freedom Index, which is brought out by the Fraser Institute,

India ranks 79th among 162 countries with 108th rank in business regulation.

The low rank in economic freedom makes it evident that India chains opportunities for

wealth creation by shackling economic freedom for its citizens.

Page 59: SIMPLYFYING IAS EXAM PREPARATION - INSIGHTSIAS · Insta 75 Days Revision Plan for UPSC Civil Services Prelims – 2020 This document is the compilation of 100 questions that are part

INSTA 75 Days REVISION PLAN for Prelims 2020 - InstaTests

www.insightsonindia.com 57 Insights IAS

56. Consider the following statements regarding Essential Commodities Act (ECA), 1955

1. It controls the production, supply and distribution of, and trade and commerce in,

certain goods such as vegetables, pulses, edible oils, sugar etc., which are treated

as essential commodities.

2. The powers to implement the provisions of the Act are delegated to the States.

3. ECA interferes in agricultural marketing by disincentivizing investments in

warehousing and storage facilities.

Which of the statements given above is/are correct?

(a) 1 and 2 only

(b) 2 and 3 only

(c) 1 and 3 only

(d) 1, 2 and 3

Solution: D

Essential Commodities Act (ECA), 1955 Features:

• Controls the production, supply and distribution of, and trade and commerce in,

certain goods such as vegetables, pulses, edible oils, sugar etc., which are treated as

essential commodities by Imposing Stockholding limit, restrict movement of goods,

and mandate compulsory purchases under the system of levy.

• The powers to implement the provisions of the Act are delegated to the States.

• The purported aim of this Act is to ensure affordability of essential commodities for

the poor by restricting hoarding.

• Impact: Creates Market Distortions: As agriculture is a seasonal activity, it is essential

to store produce for the off-season to ensure smoothened availability of a product at

stable prices throughout the year.

• Therefore, producers face an inherent tradeoff between building an inventory in the

harvest season and drawing down inventory in the lean season. ECA interferes with

this mechanism by disincentivizing investments in warehousing and storage facilities

due to frequent and unpredictable imposition of stock limits.

57. Consider the following statements regarding Green Crackers

1. The chemical formulation of green crackers produces water molecules, which

substantially reduces emission levels and absorbs dust.

2. It promises a reduction in particulate matters and harmful gases, like nitrous oxide

and sulfur oxide, by 90% per cent.

Page 60: SIMPLYFYING IAS EXAM PREPARATION - INSIGHTSIAS · Insta 75 Days Revision Plan for UPSC Civil Services Prelims – 2020 This document is the compilation of 100 questions that are part

INSTA 75 Days REVISION PLAN for Prelims 2020 - InstaTests

www.insightsonindia.com 58 Insights IAS

Which of the statements given above is/are correct?

(a) 1 only

(b) 2 only

(c) Both 1 and 2

(d) Neither 1 nor 2

Solution: A

Green Crackers

• They are less harmful and less dangerous than the conventional ones. They are the

crackers with reduced emission and decibel level.

• They are known as ‘green’ firecrackers because they have a chemical formulation

that produces water molecules, which substantially reduces emission levels and

absorbs dust.

• It promises a reduction in particulate matters and harmful gases, like nitrous oxide

and sulfur oxide, by 30- 35 per cent and the green crackers will be 25-30 per cent

cheaper to manufacture.

• Indian Council of Scientific and Industrial Research (CSIR) has developed green

crackers, which are new and improved formulations of the previous sound-emitting

crackers and other fireworks

58. Consider the following statements regarding India’s Export scenario

1. Post the 1991 reforms, India’s share in merchandise (goods) exports has

decreased at around 13 per cent per annum.

2. On an average, Network Products accounts for about 42 per cent of world

manufactured exports.

3. Network Products exports accounts for a very small share (10 per cent in 2018) in

India’s export basket.

Which of the statements given above is/are correct?

(a) 1 and 2 only

(b) 2 and 3 only

(c) 1 and 3 only

(d) 1, 2 and 3

Page 61: SIMPLYFYING IAS EXAM PREPARATION - INSIGHTSIAS · Insta 75 Days Revision Plan for UPSC Civil Services Prelims – 2020 This document is the compilation of 100 questions that are part

INSTA 75 Days REVISION PLAN for Prelims 2020 - InstaTests

www.insightsonindia.com 59 Insights IAS

Solution: B

Facts on India’s Export scenario:

• Post the 1991 reforms, India’s share in merchandise (goods) exports has grown at 13.2

per cent per annum.

• India’s share in world exports has increased from 0.6 per cent in 1991 to 1.7per cent

in 2018. Yet, even by 2018, India’s world market share remains paltry compared to

12.8 per cent for China.

• Imports of merchandise have grown faster (at the rate of 14.9 per cent per annum

during 1993-2018) than exports, resulting in increasing trade deficits.

World Exports of Network Products: Trends and Patterns

• World exports of NP increased steadily from US$ 2.01 Trillion in 2000 to US$5.41

Trillion in 2018. On an average, NP accounts for about 42 per cent of world

manufactured exports.

• Asia’s share in world exports of NP increased phenomenally from about 37 percent in

2000 to 51 per cent in 2018. East Asia accounted for the bulk of total Asian exports

followed by Southeast. Rest of Asia (including South, Central and Western Asia)

accounted for just 3 per cent of the total Asian exports.

• India’s export of NP increased from about US$2 billion in 2000 to US$32 billion in 2018,

its participation in this market remains miniscule compared to that of other Asian

countries. Despite some increase, NP exports accounts for a very small share (10 per

cent in 2018) in India’s export basket. In contrast, these products account for about

one half of the total national exports of China, Japan and Korea.

• Main category of NP exported by India is Road vehicles with a share of 4.9 percent in

its total exports in 2018 (up from 1.3 per cent in 2000). In contrast, Electrical

machinery, which accounts for the largest share in the export baskets of China (16.8

per cent) and Korea (30.5 per cent), accounts for less than 3 percent of India’s total

exports.

59. Consider the following statements regarding Nationalization of banks

1. Public sector banks (PSBs) in India were formed through two waves of

nationalizations, one in 1950 and the other in 1969.

2. After the nationalization, PSBs had a 91 per cent share in the national banking

market.

Which of the statements given above is/are correct?

(a) 1 only

(b) 2 only

(c) Both 1 and 2

Page 62: SIMPLYFYING IAS EXAM PREPARATION - INSIGHTSIAS · Insta 75 Days Revision Plan for UPSC Civil Services Prelims – 2020 This document is the compilation of 100 questions that are part

INSTA 75 Days REVISION PLAN for Prelims 2020 - InstaTests

www.insightsonindia.com 60 Insights IAS

(d) Neither 1 nor 2

Solution: B

Banking structure: Since Nationalization till today

• The modern banking system in India has its roots in the colonial era starting in the

1800s. PSBs in India were formed through two waves of nationalizations, one in 1969

and the other in 1980.

• After the 1980 nationalization, PSBs had a 91 per cent share in the national banking

market with the remaining 9 per cent held by “old private banks” (OPBs) that were

not nationalized. However, the share of PSBs has reduced to70% in recent times.

Reduced stake has been absorbed by New Private Banks (NPBs) which came up in early

1990s after liberalization.

The government exercises significant control over all aspects of PSB operations ranging from

policies on recruitment and pay to investments and financing and bank governance including

board and top management appointments.

The majority ownership by the government also subjects PSB officers to scrutiny of their

decisions by the central vigilance commission and the comptroller auditor general. With no

real restrictions on what can be investigated and under what circumstances, officers of state-

run banks are wary of taking risks in lending or in renegotiating bad debt, due to fears of

harassment under the veil of vigilance investigations.

60. Mahamastakabhisheka, a great religious event, is associated with and done for whom

of the following?

(a) Nataraja

(b) Buddha

(c) Mahavir

(d) Bahubali

Solution: D

• The Mahamastakabhisheka, refers to the abhiṣheka of the Jain images when held on

a large scale. The most famous of such consecrations is the anointment of the Bahubali

Gommateshwara Statue located at Shravanabelagola in Karnataka, India. It is an

important Jain festival held once in every 12 years. (UPSC 2009 Prelims).

Page 63: SIMPLYFYING IAS EXAM PREPARATION - INSIGHTSIAS · Insta 75 Days Revision Plan for UPSC Civil Services Prelims – 2020 This document is the compilation of 100 questions that are part

INSTA 75 Days REVISION PLAN for Prelims 2020 - InstaTests

www.insightsonindia.com 61 Insights IAS

61. Consider the following statements regarding government receipts

1. Non tax revenue forms the largest share in government receipts.

2. In the tax revenues, GST and Corporation tax accounts for largest share.

Which of the statements given above is/are correct?

(a) 1 only

(b) 2 only

(c) Both 1 and 2

(d) Neither 1 nor 2

Solution: B

Receipts Highlights for 2020-21

• Total receipts (including borrowings) in 2020-21 are estimated to be Rs30,42,230

crore and net receipts (excluding borrowings) to be Rs 22,45,893 crore. Receipts

(without borrowings) are estimated to increase by 16.3% over the revised estimates

of 2019-20.

• Gross tax revenue is budgeted to increase by 12% over the revised estimates of

2019-20, which is higher than the estimated nominal GDP growth of 10% in 2020-21.

The net tax revenue of the central government(excluding states’ share in taxes) is

estimated to be Rs 16,35,909 crore in2020-21.

Page 64: SIMPLYFYING IAS EXAM PREPARATION - INSIGHTSIAS · Insta 75 Days Revision Plan for UPSC Civil Services Prelims – 2020 This document is the compilation of 100 questions that are part

INSTA 75 Days REVISION PLAN for Prelims 2020 - InstaTests

www.insightsonindia.com 62 Insights IAS

• Devolution to states from center’s tax revenue is estimated to be Rs7,84,181 crore

in 2020-21. In 2019-20, the devolution to states reduced by19% from an estimate of

Rs 8,09,133 crore at the budgeted stage to Rs6,56,046 crore at the revised stage.

• Non-tax revenue is expected to be Rs 3,85,017 crore in 2020-21. This is 11.4% higher

than the revised estimate of 2019-20.

• Capital receipts (without borrowings) are budgeted to increase by 175.7%over the

revised estimates of 2019-20. This is on account of disinvestments, which are expected

to be Rs 2,10,000 crore in 2020-21, as compared to Rs65,000 crore as per the revised

estimates of 2019-20.

62. Consider the following statements regarding subsidies in India

1. Subsidies account for around 24% of the total expenditure by government.

2. The expenditure on major subsidies is around 1.4 per cent of GDP in 2019-20.

3. Of the total subsidies, Food subsidies account for the largest share.

Which of the statements given above is/are correct?

(a) 1 and 2 only

(b) 2 and 3 only

(c) 1 and 3 only

(d) 1, 2 and 3

Solution: B

Page 65: SIMPLYFYING IAS EXAM PREPARATION - INSIGHTSIAS · Insta 75 Days Revision Plan for UPSC Civil Services Prelims – 2020 This document is the compilation of 100 questions that are part

INSTA 75 Days REVISION PLAN for Prelims 2020 - InstaTests

www.insightsonindia.com 63 Insights IAS

• The expenditure on major subsidies, which is a significant component of non

committed revenue expenditure was pegged at 1.4 per cent of GDP in 2019-20 BE.

The budgetary expenditure on major subsidies has shown a declining trend over the

past years.

Expenditure on Subsidies

In 2020-21, the total expenditure on subsidies is estimated to be Rs 2,62,109 crore, a decrease

of 0.5% from the revised estimate of 2019-20. This is largely due to a decrease in expenditure

on fertilizer subsidy. Details are given below:

• Food subsidy: Allocation to food subsidy is estimated at Rs 1,15,570 crore in 2020-

21, a 6.3% increase as compared to the revised estimate of2019-20. In 2019-20

budget, Rs 1,84,220 crore was allocated to food subsidy. However, the revised

estimate is much lower than the budgeted estimate at Rs 1,08,688 crore. This is due

to a 41% cut (Rs 75,532 crore in amount) in the allocation to food subsidy for 2019-20

from the budgeted stage to the revised stage.

• Fertilizer subsidy: Expenditure on fertilizer subsidy is estimated at Rs71,309 crore in

2020-21. This is a decrease of Rs 8,689 crore (10.9%) from the revised estimate of

2019-20.

• Petroleum subsidy: Expenditure on petroleum subsidy is estimated to increase by

6.1% to Rs 40,915 crore in 2020-21. Petroleum subsidy consists of subsidy on LPG (Rs

37,256 crore) and kerosene subsidy (Rs 3,659 crore). In 2020-21, while the LPG subsidy

is estimated to increase by Rs 3,170crore over the previous year, kerosene subsidy is

estimated to decrease by Rs 824 crore.

• Other subsidies: Expenditure on other subsidies includes interest subsidies for

various government schemes, subsidies for the price support scheme for agricultural

produce, and assistance to state agencies for procurement, among others. In 2020-

21, the expenditure on these other subsidies has decreased by Rs 1,987 crore (5.5%)

over the revised estimate of 2019-20. Table 4 provides details of subsidies in 2020-21.

Page 66: SIMPLYFYING IAS EXAM PREPARATION - INSIGHTSIAS · Insta 75 Days Revision Plan for UPSC Civil Services Prelims – 2020 This document is the compilation of 100 questions that are part

INSTA 75 Days REVISION PLAN for Prelims 2020 - InstaTests

www.insightsonindia.com 64 Insights IAS

63. Consider the following statements regarding National e-Vidhan Application (NeVA)

Project.

1. It is a Mission Mode Project (MMP) comes under the Digital India Programme.

2. The Ministry of Electronics and Information Technology (MietY) is Nodal Ministry’

for its implementation of the programme.

3. It aims to bring all the legislatures of the country together by technology.

Which of the statements given above is/are correct?

(a) 1 only

(b) 2 and 3 only

(c) 1 and 3 only

(d) 1, 2 and 3

Solution: C

National e-Vidhan Application (NeVA) Project:

• Minister of State for Parliamentary Affairs and Water Resources, River Development

and Ganga Rejuvenation, gave details on the status of implementation of National e-

Vidhan Application (NeVA) Project.

e-Vidhan:

• It is a Mission Mode Project (MMP) comes under the Digital India Programme.

• Ministry of Parliamentary Affairs (MoPA) is the ‘Nodal Ministry’ for its

implementation in all the 31 States/UTs with Legislatures.

• The funding for e-Vidhan is provided by the MoPA and technical support by Ministry

of Electronics and Information Technology (MietY).

• The funding of NeVA is through Central Sponsored Scheme i.e. 60:40; and 90:10 for

North East & hilly States and 100% for UTs.

Aim of NeVA:

• It aims to bring all the legislatures of the country together, in one platform thereby

creating a massive data depository without having the complexity of multiple

applications.

• Paperless Assembly or e-Assembly is a concept involving of electronic means to

facilitate the work of Assembly.

• It enables automation of entire law making process, tracking of decisions and

documents, sharing of information.

• Through the cloud technology (Meghraj), data deployed can be accessed anywhere

at any time.

Page 67: SIMPLYFYING IAS EXAM PREPARATION - INSIGHTSIAS · Insta 75 Days Revision Plan for UPSC Civil Services Prelims – 2020 This document is the compilation of 100 questions that are part

INSTA 75 Days REVISION PLAN for Prelims 2020 - InstaTests

www.insightsonindia.com 65 Insights IAS

• The live webcasting of Lok Sabha TV and Rajya Sabha TV is also available on this

application.

• Doordarshan has already been enabled with provision to incorporate similar facility in

respect of State Legislatures.

• Himachal Pradesh is already the first Digital Legislature of the country.

64. Consider the following statements regarding import basket

1. China continues to be the largest exporter to India followed by USA, UAE and

Saudi Arabia.

2. In the import basket of 2019-20, petroleum products had the largest share.

3. Between 2011-12 and 2019-20, imports of Electronics grew the fastest from a

negligible share to 3.6 per cent of import basket.

Which of the statements given above is/are correct?

(a) 1 and 2 only

(b) 2 and 3 only

(c) 1 and 3 only

(d) 1, 2 and 3

Solution: C

Page 68: SIMPLYFYING IAS EXAM PREPARATION - INSIGHTSIAS · Insta 75 Days Revision Plan for UPSC Civil Services Prelims – 2020 This document is the compilation of 100 questions that are part

INSTA 75 Days REVISION PLAN for Prelims 2020 - InstaTests

www.insightsonindia.com 66 Insights IAS

• In the import basket of 2019-20 (April-November), crude petroleum had the largest

share followed by gold and petroleum products. However, between2011-12 and 2019-

20, imports of Electronics grew the fastest from a negligible share to 3.6 per cent

(Figure 15).

• China continues to be the largest exporter to India followed by USA, UAE and Saudi

Arabia. In recent times, Hong Kong, Korea and Singapore have also emerged as

significant exporters to India (Figure 16).

65. With which of the following countries does India have trade deficit?

1. China

2. Saudi Arabia

3. UAE

4. Germany

Select the correct answer using the code given below:

(a) 1, 2 and 3 only

(b) 2, 3 and 4 only

(c) 1, 2 and 4 only

(d) 1, 2, 3 and 4

Solution: C

Page 69: SIMPLYFYING IAS EXAM PREPARATION - INSIGHTSIAS · Insta 75 Days Revision Plan for UPSC Civil Services Prelims – 2020 This document is the compilation of 100 questions that are part

INSTA 75 Days REVISION PLAN for Prelims 2020 - InstaTests

www.insightsonindia.com 67 Insights IAS

• The bilateral trade position with respect to top trading partners over a period of time

is shown in Table 3. With two top trading countries i.e. USA and United Arab Emirates,

India has consistently run trade surplus since 201415. On the other hand, India has

trade deficit continuously since 2014-15with respect to other major trading partners

i.e. China PRP, Saudi Arabia, Iraq, Germany, Korea RP, Indonesia and Switzerland. India

had trade surplus with Hong Kong and Singapore till 2017-18, before it changed to

trade deficit in 2018-19. The bilateral imbalances have remained stable in most cases.

66. Consider the following statements regarding Institutions of Eminence Scheme

1. This has been launched by University Grants Commission (UGC).

2. Under the scheme, there will be financial assistance to the private institutions

based on their performance.

3. Under the scheme, selected institutions will have greater autonomy compared to

other higher education institutions.

Which of the statements given above is/are correct?

(a) 1 only

(b) 2 and 3 only

(c) 1 and 3 only

(d) 1, 2 and 3

Solution: C

Page 70: SIMPLYFYING IAS EXAM PREPARATION - INSIGHTSIAS · Insta 75 Days Revision Plan for UPSC Civil Services Prelims – 2020 This document is the compilation of 100 questions that are part

INSTA 75 Days REVISION PLAN for Prelims 2020 - InstaTests

www.insightsonindia.com 68 Insights IAS

Institutions of Eminence scheme:

• The scheme of IoE was rolled out by University Grants Commission (UGC).

• It aims to project Indian institutes to global recognition.

• The selected institutes will enjoy complete academic and administrative autonomy.

• The selection shall be made through challenge method mode by the Empowered

Expert Committee constituted for the purpose.

• Eligibility: Only higher education institutions currently placed in the top 500 of global

rankings or top 50 of the National Institutional Ranking Framework (NIRF) are eligible

to apply for the eminence tag.

• The private Institutions of Eminence can also come up as Greenfield ventures-

provided the sponsoring organisation submits a convincing perspective plan for 15

years.

• There will be no financial assistance to the private institutions.

67. Consider the following statements regarding Net Services

1. An increase in service exports to GDP ratio has a net positive impact on the BoP

position.

2. Over the years, service imports in relation to GDP has been steadily rising.

3. India’s net services surplus has been steadily increasing in relation to GDP.

Which of the statements given above is/are correct?

(a) 1 and 2 only

(b) 2 and 3 only

(c) 1 and 3 only

(d) 1, 2 and 3

Solution: A

Net Services: India’s net services surplus has been steadily declining in relation to GDP since

it reached its peak to about two-thirds of merchandise deficit in 2016-17.

Services Exports: An increase in service exports to GDP ratio has a net positive impact on the

BoP position.

• India’s service exports have consistently hovered between 7.4 to 7.7per cent of GDP.

• Software services constitute around 40-45 per cent, followed by business services at

about 18-20 per cent, travel at 11-14 per cent and transportation at 9-11 per cent of

service exports.

Page 71: SIMPLYFYING IAS EXAM PREPARATION - INSIGHTSIAS · Insta 75 Days Revision Plan for UPSC Civil Services Prelims – 2020 This document is the compilation of 100 questions that are part

INSTA 75 Days REVISION PLAN for Prelims 2020 - InstaTests

www.insightsonindia.com 69 Insights IAS

Service Imports: An increase in service imports to GDP ratio has a net negative impact on the

BoP position.

• Over the years, service imports in relation to GDP has been steadily rising given arising

level of FDI and a gradual up scaling of the Make in India program.

• Business services constitute about a third of service imports and the component of

travel services has been steadily increasing reflecting the growing attractiveness of

global destinations to the domestic tourists in the country.

68. Consider the following statements regarding Net Remittances

1. The Migration Report 2019 released by the United Nations has placed India as the

leading country of origin of international migrants in 2019.

2. According to a report of World Bank, India remained the top remittance recipient

country in 2018 followed by China.

Which of the statements given above is/are correct?

(a) 1 only

(b) 2 only

(c) Both 1 and 2

(d) Neither 1 nor 2

Solution: C

Net Remittances: An increase in net remittances improves the BoP position.

• Net remittances from Indians employed overseas has increased with the amount

received in the first half of 2019-20 being more than 50per cent of the total receivables

in 2018-19.

• The Migration Report 2019 released by the United Nations has placed India as the

leading country of origin of international migrants in 2019 with a Diaspora strength of

17.5 million.

• Further, as per the October 2019 report of World Bank, India remained the top

remittance recipient country in 2018 followed by China.

69. Consider the following statements regarding Price stabilization fund

1. It has been launched under APMC Act

2. It is Central Sector Scheme

3. It aims to support market interventions for price control of perishable agri-

horticultural commodities.

Page 72: SIMPLYFYING IAS EXAM PREPARATION - INSIGHTSIAS · Insta 75 Days Revision Plan for UPSC Civil Services Prelims – 2020 This document is the compilation of 100 questions that are part

INSTA 75 Days REVISION PLAN for Prelims 2020 - InstaTests

www.insightsonindia.com 70 Insights IAS

Which of the statements given above is/are correct?

(a) 1 and 2 only

(b) 2 and 3 only

(c) 1 and 3 only

(d) 1, 2 and 3

Solution: B

Price stabilization fund:

• It is Central Sector Scheme

• Price Stabilization Fund (PSF) refers to any fund constituted for the purpose of

containing extreme volatility in prices of selected commodities

• The amount in the fund is generally utilized for activities aimed at bringing down/up

the high/low prices say for instance, procurement of such products and distribution

of the same as and when required, so that prices remain in a range.

• Many countries use such dedicated funds for stabilization of major petroleum product

prices, particularly if they are importers.

• Some countries use such funds for stabilizing not just commodity prices but a variety

of key macroeconomic variables such as the exchange rate (which is nothing but the

price of the domestic currency expressed in terms of an external currency),

benchmark stock indices etc.

• It aims to support market interventions for price control of perishable agri-

horticultural commodities.

70. Which of the following is/are the reasons for downward trend in yield on G-Sec?

1. Change in monetary policy stance of the U.S. Fed.

2. Easing of liquidity condition of the banking system.

3. Transfer of RBI surplus to the Government.

4. Significant and sustained surplus liquidity.

Select the correct answer using the code given below:

(a) 1, 2 and 3 only

(b) 2, 3 and 4 only

(c) 1, 3 and 4 only

(d) 1, 2, 3 and 4

Page 73: SIMPLYFYING IAS EXAM PREPARATION - INSIGHTSIAS · Insta 75 Days Revision Plan for UPSC Civil Services Prelims – 2020 This document is the compilation of 100 questions that are part

INSTA 75 Days REVISION PLAN for Prelims 2020 - InstaTests

www.insightsonindia.com 71 Insights IAS

Solution: D

Developments in G-sec Market: Initially yield on G-Sec hardened marginally on account of

rise in crude oil prices.

Thereafter, it largely followed a downward trend which may be attributed to various reasons:

• Change in monetary policy stance of the U.S. Fed (on global growth concerns and

ongoing trade tensions).

• Easing of liquidity condition of the banking system.

• Consecutive policy rate cuts by the RBI along with change of stance from neutral to

accommodative.

• Benign crude oil prices aided the sentiment.

• Transfer of RBI surplus to the Government.

• Significant and sustained surplus liquidity.

• Special Open Market Operation (purchase of long-term securities and simultaneous

sale of short-term securities) by Reserve Bank of India also helped bring down the

yield slightly on 10-year G-Secs.

71. Consider the following statements regarding PRAGATI platform.

1. It has been launched by Department of Administrative Reforms and Public

Grievances

2. It is multi-modal platform for Pro-Active Governance and Timely Implementation

of programmes and projects

Which of the statements given above is/are correct?

(a) 1 only

(b) 2 only

(c) Both 1 and 2

(d) Neither 1 nor 2

Solution: B

PRAGATI (Pro-Active Governance and Timely Implementation):

• The Prime Minister recently chaired his thirtieth interaction through PRAGATI – the

ICT-based, multi-modal platform for Pro-Active Governance and Timely

Implementation.

Page 74: SIMPLYFYING IAS EXAM PREPARATION - INSIGHTSIAS · Insta 75 Days Revision Plan for UPSC Civil Services Prelims – 2020 This document is the compilation of 100 questions that are part

INSTA 75 Days REVISION PLAN for Prelims 2020 - InstaTests

www.insightsonindia.com 72 Insights IAS

• PRAGATI (Pro-Active Governance and Timely Implementation) is a unique

integrating and interactive platform.

• The platform is aimed at addressing common man’s grievances, and simultaneously

monitoring and reviewing important programmes and projects of the Government of

India as well as projects flagged by State Governments.

• The PRAGATI platform uniquely bundles three latest technologies: Digital data

management, video-conferencing and geo-spatial technology.

• It also offers a unique combination in the direction of cooperative federalism since it

brings on one stage the Secretaries of Government of India and the Chief Secretaries

of the States.

• With this, the Prime Minister is able to discuss the issues with the concerned Central

and State officials with full information and latest visuals of the ground level situation.

It is also an innovative project in e-governance and good governance.

• It is a three-tier system (PMO, Union Government Secretaries, and Chief Secretaries

of the States).

• Issues to be flagged before the PM are picked up from the available database

regarding Public Grievances, on-going Programmes and pending Projects.

72. Consider the following statements regarding India’s progress towards SDGs

1. NITI Aayog has come up with a single measurable SDG index to track the progress

of all the States and UTs.

2. The SDG score varies from 0 to 100. A score of 100 implies that the States/UTs

have achieved the targets set for 2030.

3. India’s growth trajectory for achieving SDG 10 (Reduced Inequality) and SDG 15

(Life on Land) is impressive as compared to the other SDGs.

Which of the statements given above is/are correct?

(a) 1 and 2 only

(b) 2 and 3 only

(c) 1 and 3 only

(d) 1, 2 and 3

Solution: D

India’s progress towards SDGs

• NITI Aayog has come up with a single measurable SDG index to track the progress of

all the States and UTs across 13 out of 17 SDGs (excluding Goal 12, 13, 14 and 17 on

Page 75: SIMPLYFYING IAS EXAM PREPARATION - INSIGHTSIAS · Insta 75 Days Revision Plan for UPSC Civil Services Prelims – 2020 This document is the compilation of 100 questions that are part

INSTA 75 Days REVISION PLAN for Prelims 2020 - InstaTests

www.insightsonindia.com 73 Insights IAS

account of unavailability of comparable data across States/UTs), it helps in informed

policy formulations.

• The SDG score varies from 0 to 100. A score of 100 implies that the States/UTs have

achieved the targets set for 2030; a score of 0implies that the particular State/UT is at

the bottom of the table.

• States with scores equal to/greater than 65 are considered as Front-Runners; in the

range of 50-64 as Performers and as Aspirants if the score is less than 50. Kerala and

Himachal Pradesh are the frontrunners amongst all the states. It is noteworthy that

none of the States/ UTs fall in the Aspirant category in 2019.

• Success – India’s growth trajectory for achieving SDG 10 (Reduced Inequality) and

SDG 15 (Life on Land) is impressive as compared to the other SDGs as several states

have achieved 100 in these SDGs. This may be due to performance in worthy initiatives

such as PMJDY, Mahatma Gandhi National Rural Employment Guarantee Act

(MGNREGA), the National Environment Policy, National Agro-forestry Policy and

Green Highways Policy.

• Struggling – India is struggling to achieve its targets of SDG 5 (Gender Equality) and

SDG. 11 (Sustainable Cities and Communities) as large number of states are in the

‘Aspirants’ category’. Goa, a front runner among all the States and UTs in SDG 11, has

been doing exceptionally well in waste management.

73. The Kandangi saree recently got GI tag, belongs to the state of:

(a) Karnataka

(b) Tamil Nadu

(c) Maharashtra

(d) Andhra Pradesh

Solution: B

GI Atal for Dindigul lock and Kandangi saree:

Two products from Tamil Nadu — Dindigul lock and Kandangi Saree — have been given the

Geographical Indication (GI) tag by The Geographical Indications Registry in Chennai.

1. The Dindigul locks are known throughout the world for their superior quality and

durability, so much so that even the city is called Lock City.

2. The Kandangi sarees are manufactured in the entire Karaikudi taluk in Sivaganga

district. They are characterized by large contrast borders and some are known to have

borders covering as far as two-thirds of the saree which is usually around 5.10 m-5.60

m in length.

Page 76: SIMPLYFYING IAS EXAM PREPARATION - INSIGHTSIAS · Insta 75 Days Revision Plan for UPSC Civil Services Prelims – 2020 This document is the compilation of 100 questions that are part

INSTA 75 Days REVISION PLAN for Prelims 2020 - InstaTests

www.insightsonindia.com 74 Insights IAS

74. Consider the following statements regarding Food processing sector

1. The sector constituted as much as 8.83 per cent and 10.66 per cent of GVA in

Manufacturing and Agriculture sector respectively.

2. The value of processed food exports during 2018-19 accounts for about 10 per

cent of India’s total exports.

3. The sector has been growing at an average annual growth rate of around 15 per

cent.

Which of the statements given above is/are correct?

(a) 1 and 2 only

(b) 2 and 3 only

(c) 1 and 3 only

(d) 1, 2 and 3

Solution: A

Food processing sector

• A well-developed food processing sector with higher level of processing helps in the

reduction of wastage, improves value addition, promotes crop diversification, ensures

better return to the farmers, promotes employment as well as increases export

earnings. Growth in the food processing sector is also expected to open up

opportunities for players having strong linkages in the agri-value chain. During the last

6 years ending 2017-18, Food Processing Industries sector has been growing at an

average annual growth rate of around 5.06 per cent. The sector constituted as much

as 8.83 per cent and 10.66 per cent of GVA in Manufacturing and Agriculture sector

respectively in 2017-18 at 2011-12 prices. 7.32 According to the Annual Survey of

Industries for 2016-17, the total number of persons engaged in registered food

processing sector was 18.54 lakhs. 7.33 Unregistered food processing sector supports

employment to 51.11 lakh workers as per the NSSO 73rd Round, 2015-16 and

constitutes 14.18 per cent of employment in the unregistered manufacturing sector.

The value of processed food exports during 2018-19 was of the order of US $ 35.30

billion accounting for about 10.70 per cent of India’s total exports (total exports US $

330.08 billion). The value of import of processed food during 2018-19 was US $ 19.32

billion which is 3.76 per cent of India’s total imports.

75. Advance Pricing Agreements (APAs), sometime seen in the news, is related to

(a) Taxation

(b) Allocation of spectrum and charges on consumer

Page 77: SIMPLYFYING IAS EXAM PREPARATION - INSIGHTSIAS · Insta 75 Days Revision Plan for UPSC Civil Services Prelims – 2020 This document is the compilation of 100 questions that are part

INSTA 75 Days REVISION PLAN for Prelims 2020 - InstaTests

www.insightsonindia.com 75 Insights IAS

(c) Mine leasing agreement

(d) Solar energy purchase by DISCOMS

Solution: A

• An advance pricing agreement (APA) is an ahead-of-time agreement between a

taxpayer and a tax authority on an appropriate transfer pricing methodology (TPM)

for a set of transactions at issue over a fixed period of time (called “Covered

Transactions”).

https://economictimes.indiatimes.com/news/economy/finance/cbdt-inks-300th-advance-

pricing-agreement/articleshow/71395453.cms?from=mdr

DAY – 68 (InstaTest-68)

76. Consider the following statements regarding Satnami sect

1. Satnami sect was founded by a saint named “Birbhan” in 1657 in Narnaul in

Haryana.

2. The followers of this sect kept their heads shaven and abstained from liquor and

meat.

Which of the statements given above is/are correct?

(a) 1 only

(b) 2 only

(c) Both 1 and 2

(d) Neither 1 nor 2

Solution: C

• The Satnami sect was founded by a saint named “Birbhan” in 1657 in Narnaul in

Haryana. The major religious activity of this sect is to chant and meditate the true

names {Sat-Nam} of God specially Rama and Krishna.

• The followers of this sect kept their heads shaven (thus called Mundiyas) and

abstained from liquor and meat.

Page 78: SIMPLYFYING IAS EXAM PREPARATION - INSIGHTSIAS · Insta 75 Days Revision Plan for UPSC Civil Services Prelims – 2020 This document is the compilation of 100 questions that are part

INSTA 75 Days REVISION PLAN for Prelims 2020 - InstaTests

www.insightsonindia.com 76 Insights IAS

Satnami Rebellion

• The revolt triggered when a Mughal soldier killed a Satnami. The Satnamis killed the

soldier in revenge and in turn Mughal soldiers were sent to teach them a lesson. Some

5,000 Satnamis stood up in arms and routed the Mughal troops in the town, drove

away the Mughal administrators and set up their own administration under their

leader Birbhan.

77. Consider the following statements regarding emission standards in India

1. Vehicle emission norms were introduced in India in 1991 for petrol and in 1992

for diesel vehicles.

2. Euro norms are followed in India under the name Bharat Stage Emission

Standards.

3. The standards and the timeline for implementation are set by the Central

Pollution Control Board.

Which of the statements given above is/are correct?

(a) 1 and 2 only

(b) 2 and 3 only

(c) 1 and 3 only

(d) 1, 2 and 3

Solution: D

Definition of ‘Bharat Emission Standards’

Definition: These are the standards set up by the Indian government which specify the

amount of air pollutants from internal combustion engines, including those that vehicles can

emit. If these emit more pollutants than the prescribed limit, they don’t get a clearance to be

sold in an open market. Bharat Stage Emission Standards have been instituted by the Central

Pollution Control Board (CPCB), instituted within the Ministry of Environment Forests and

Climate Change.

Description: Vehicle emission norms were introduced in India in 1991 for petrol and in 1992

for diesel vehicles. Since 2000, Euro norms are followed in India under the name Bharat Stage

Emission Standards for four wheeled vehicles. Bharat stage III norms have been enforced

across India since October 2010. In a few cities, Bharat stage IV norms are in place since April

2010. Bharat stage IV is proposed to be enforced throughout India by April 2017. It has already

been put into use in 13 major cities.

Page 79: SIMPLYFYING IAS EXAM PREPARATION - INSIGHTSIAS · Insta 75 Days Revision Plan for UPSC Civil Services Prelims – 2020 This document is the compilation of 100 questions that are part

INSTA 75 Days REVISION PLAN for Prelims 2020 - InstaTests

www.insightsonindia.com 77 Insights IAS

Upgrading the emission norms requires the manufacturing companies to upgrade their

technology, which in turn increases the cost of the vehicle. Cost is one of the main reasons

for the slow upgrade of emission standards. However, there are also arguments that the

increase in cost is made up by savings in health costs as the pollutants causing diseases are

decreased with the upgrade in emission standards. Fuels also play a crucial role in meeting

these emission norms. Fuel specifications have also been aligned to its corresponding

European production norms.

• It is to regulate the output of air pollutants from internal combustion engines and

Spark-ignition engines equipment, including motor vehicles.

• It includes both emission standards for new vehicles as well as specifications for

commercial petrol and diesel fuels.

• The standards and the timeline for implementation are set by the Central Pollution

Control Board under the Ministry of Environment & Forests and climate change.

• All new vehicles manufactured after the implementation of BS norms must be

compliant with the regulations.

• Major emissions governed under these norms are carbon monoxide emissions,

hydrocarbon emission limits. Nitrogen Oxides and particulate matter are also

significant metrics.

• BS-IV standard was brought into place in country-wide in April 2017.

78. Consider the following statements regarding Taj Trapezium Zone (TTZ)

1. It is a defined area of 10,400 sq km around the Taj Mahal to protect the

monument from pollution.

2. The TTZ comprises only two World Heritage Sites the Taj Mahal and Agra Fort.

Which of the statements given above is/are correct?

(a) 1 only

(b) 2 only

(c) Both 1 and 2

(d) Neither 1 nor 2

Solution: A

• Taj Trapezium Zone (TTZ) is a defined area of 10,400 sq km around the Taj Mahal to

protect the monument from pollution. The Supreme Court of India delivered a ruling

on December 30, 1996 regarding industries covered under the TTZ, in response to a

PIL seeking to protect the Taj Mahal from environmental pollution. It banned the use

of coal/ coke in industries located in the TTZ with a mandate for switching over from

Page 80: SIMPLYFYING IAS EXAM PREPARATION - INSIGHTSIAS · Insta 75 Days Revision Plan for UPSC Civil Services Prelims – 2020 This document is the compilation of 100 questions that are part

INSTA 75 Days REVISION PLAN for Prelims 2020 - InstaTests

www.insightsonindia.com 78 Insights IAS

coal/ coke to natural gas, and relocating them outside the TTZ or shutting down. The

TTZ comprises monuments including three World Heritage Sites the Taj Mahal, Agra

Fort and Fatehpur Sikri. TTZ is so named since it is located around the Taj Mahal and

is shaped like a trapezoid.

• By the order of Ministry of Environment and Forests in exercise of the powers

conferred by the Environment Protection Act, 1986 and in supersession of the order

of the government of India in the ministry of Environment and forests, number S.O.

350(E), dated 17th May, 1999, except as thins done or omitted to be done before such

supersession, the central Government hereby re-constitutes the Taj Trapezium Zone

Pollution (Prevention and Control) authority.

Area Under TTZ

• The geographical limits of the Taj Trapezium Zone is defined in the shape of trapezoid

between 26° 45N & 77 ° 15E to 27 ° 45N & 77° 15E in the West of the Taj Mahal and

in the East of Taj Mahal between 27° 00N & 78° 30E to 27° 30E, lying in the Agra

Division of the State of Uttar Pradesh and in the Bharatpur Division of the State of

Rajasthan.

Responsibility areas :

• Protecting Taj Mahal from damage via pollution

• Control Pollution

• Keep record of Taj Mahal pollution statics

• Study city pollution

79. Global competitiveness index, sometime seen in the news, is published by

(a) OECD

(b) World Bank

(c) AIIB

(d) World Economic Forum

Solution: D

• Global Competitiveness Index released by World Economic Forum

• The Global Competitiveness Index (GCI), which was launched in 1979, maps the

competitiveness landscape of 141 economies through 103 indicators organised into

12 pillars.

Page 81: SIMPLYFYING IAS EXAM PREPARATION - INSIGHTSIAS · Insta 75 Days Revision Plan for UPSC Civil Services Prelims – 2020 This document is the compilation of 100 questions that are part

INSTA 75 Days REVISION PLAN for Prelims 2020 - InstaTests

www.insightsonindia.com 79 Insights IAS

• Compared to last year, India has moved down 10 places to rank 68th. India was ranked

58th last year. It is among the worst-performing BRICS nations along with Brazil,

ranked even lower than India at 71st this year.

80. Consider the following statements regarding Categories of biofuels

1. First-generation biofuels are made from sugar, starch, vegetable oil, or animal fats

using conventional technology.

2. Second generation biofuels are produced from non-food crops, such as cellulosic

biofuels and waste biomass.

3. Third generation biofuels are aimed at producing sustainable energy and also

capturing and storing carbon dioxide.

Which of the statements given above is/are correct?

(a) 1 and 2 only

(b) 2 only

(c) 1 and 3 only

(d) 1, 2 and 3

Solution: A

Biofuels are liquid or gaseous fuels primarily produced from biomass, and can be used to

replace or can be used in addition to diesel, petrol or other fossil fuels for transport,

stationary, portable and other applications. Crops used to make biofuels are generally either

high in sugar (such as sugarcane, sugar beet, and sweet sorghum), starch (such as maize and

tapioca) or oils (such as soybean, rapeseed, coconut, sunflower).

Categories of biofuels

Biofuels are generally classified into three categories. They are

1. First generation biofuels – First-generation biofuels are made from sugar, starch,

vegetable oil, or animal fats using conventional technology. Common first-generation

biofuels include Bioalcohols, Biodiesel, Vegetable oil, Bioethers, Biogas.

2. Second generation biofuels – These are produced from non-food crops, such as

cellulosic biofuels and waste biomass (stalks of wheat and corn, and wood). Examples

include advanced biofuels like biohydrogen, biomethanol.

3. Third generation biofuels – These are produced from micro-organisms like algae.

Fourth Generation Bio fuels

• Aimed at producing sustainable energy and also capturing and storing carbon dioxide.

• Carbon dioxide is captured which can be then geo-sequestered.

Page 82: SIMPLYFYING IAS EXAM PREPARATION - INSIGHTSIAS · Insta 75 Days Revision Plan for UPSC Civil Services Prelims – 2020 This document is the compilation of 100 questions that are part

INSTA 75 Days REVISION PLAN for Prelims 2020 - InstaTests

www.insightsonindia.com 80 Insights IAS

• This is carbon neutral technology.

81. Consider the following statements regarding Gandhi Solar Park

1. It has been setup at Sabarmati Ashram.

2. The solar panels are powered up to reach the max of 50 KW of generation power.

3. As a symbolic effort, one panel at the park is dedicated each for every 193 UN

Member State.

Which of the statements given above is/are correct?

(a) 1 and 2 only

(b) 2 and 3 only

(c) 3 only

(d) 1, 2 and 3

Solution: B

• In a first of its kind symbolic effort by India at the UN, Prime Minister Narendra Modi

will inaugurate a 50KW ‘Gandhi Solar Park’ next week during his visit to the world

organization, a gesture that highlights India’s willingness to go beyond the talk on

climate change.

• At a contribution of about one million dollars, India has gifted solar panels that have

been installed on the roof of the UN Headquarters here, one panel each for every 193

UN Member State.

• Modi will remotely inaugurate the solar park at the UN Headquarters and the ‘Gandhi

Peace Garden’ during a special commemorative event on September 24 marking

Gandhi’s 150th birth anniversary. On the occasion, a special UN Postage on Gandhi’s

150 years will also be released.

• The ‘Gandhi Peace Garden’ is an innovative initiative under which the Consulate

General of India in New York, Long Island-based NGO Shanti Fund and the State

University of New York – Old Westbury have entered into an agreement to plant 150

trees. The garden is dedicated in the memory of Gandhi and is a crowdsourced project,

with people adopting trees in the memory of their loved ones. The garden is in an

open site within the 600 acres campus of the University.

• The solar panels are powered up to reach the max of 50 KW of generation power.

Page 83: SIMPLYFYING IAS EXAM PREPARATION - INSIGHTSIAS · Insta 75 Days Revision Plan for UPSC Civil Services Prelims – 2020 This document is the compilation of 100 questions that are part

INSTA 75 Days REVISION PLAN for Prelims 2020 - InstaTests

www.insightsonindia.com 81 Insights IAS

82. Consider the following statements regarding Mahatma Gandhi National Fellowship

Programme

1. It aims to address the challenge of non-availability of personnel for

implementation of various programmes at national, state and district levels

2. It aims to increase the number of personnel available for MNREGA works

3. It is designed under the Ministry of Rural Development

Which of the statements given above is/are correct?

(a) 1 and 2 only

(b) 1 and 3 only

(c) 2 only

(d) 1 only

Solution: D

Mahatma Gandhi National Fellowship Programme

• Ministry of Skill Development and Entrepreneurship launches Mahatma Gandhi

National Fellowship Programme with IIM Bangalore. It is a two-year fellowship

programme to boost skill development at the district level.

• Designed under Skills Acquisition and Knowledge Awareness for Livelihood

Promotion (SANKALP). It aims to address the challenge of non-availability of

personnel for implementation of various programmes at national, state and district

levels

• Eligibility: Eligible fellows for the programme have to be in 21-30 years age-group,

have a graduation degree from a recognized university and be citizens of India.

Proficiency in official language of state of fieldwork will be mandatory.

• Fellows will receive a stipend of Rs. 50,000 in the first year and Rs. 60,000 in the

second year

83. Consider the following statements regarding Bamboonomics

1. It was launched by Ministry of Environment, Forest and Climate Change.

2. It is one of the tribal movement, to promote tribal enterprise.

3. It was launched for combating desertification and the climate change at COP 14

to the UNCCD.

Which of the statements given above is/are correct?

(a) 2 only

(b) 2 and 3 only

Page 84: SIMPLYFYING IAS EXAM PREPARATION - INSIGHTSIAS · Insta 75 Days Revision Plan for UPSC Civil Services Prelims – 2020 This document is the compilation of 100 questions that are part

INSTA 75 Days REVISION PLAN for Prelims 2020 - InstaTests

www.insightsonindia.com 82 Insights IAS

(c) 1 and 3 only

(d) 1, 2 and 3

Solution: B

Bamboonomics

• Ministry of Tribal Affairs launched the biggest Tribal movement, to promote tribal

enterprise through ‘Bamboonomics’.

• It was launched for combating desertification and the climate change at COP 14 to

the UNCCD.

• TRIFED introduced the ‘The 4P1000 Initiative: The Tribal Perspective through

Bamboonomics’.

• The international initiative “4 per 1000” was launched by France in 2015 at the COP

21.

The aim of the initiative is to,

• Demonstrate that agriculture, and in particular agricultural soils can play a crucial role

where food security and climate change are concerned.

• Encourage stakeholders to transition towards a productive, highly resilient

agriculture, based on the appropriate management of lands and soils, creating jobs

and incomes hence ensuring sustainable development.

• An annual growth rate of 0.4% in the soil carbon stocks, or 4‰ per year, in the first

30-40 cm of soil, would significantly reduce the CO2 concentration in the atmosphere

related to human activities.

• This is what the 4 per 1000 Initiative proposes, soils for food security and climate.

• The initiative is intended to complement those necessary efforts to reduce

greenhouse gas emissions globally.

• It is voluntary, it is up to each member to define how they want to contribute to the

goals.

84. Consider the following statements regarding International Seed Treaty

1. It was adopted by Conference of the Food and Agriculture Organization of the

United Nations.

2. It aims at guaranteeing food security through the conservation, exchange and

sustainable use of the world’s plant genetic resources for food and agriculture

(PGRFA).

Which of the statements given above is/are correct?

(a) 1 only

Page 85: SIMPLYFYING IAS EXAM PREPARATION - INSIGHTSIAS · Insta 75 Days Revision Plan for UPSC Civil Services Prelims – 2020 This document is the compilation of 100 questions that are part

INSTA 75 Days REVISION PLAN for Prelims 2020 - InstaTests

www.insightsonindia.com 83 Insights IAS

(b) 2 only

(c) Both 1 and 2

(d) Neither 1 nor 2

Solution: C

The International Treaty on Plant Genetic Resources for Food and Agriculture was adopted

by the Thirty-First Session of the Conference of the Food and Agriculture Organization of the

United Nations on 3 November 2001.

The Treaty aims at:

• recognizing the enormous contribution of farmers to the diversity of crops that feed

the world;

• establishing a global system to provide farmers, plant breeders and scientists with

access to plant genetic materials;

• ensuring that recipients share benefits they derive from the use of these genetic

materials with the countries where they have been originated.

Main Provisions:

Multilateral system

• The Treaty’s truly innovative solution to access and benefit sharing, the Multilateral

System, puts 64 of our most important crops – crops that together account for 80

percent of the food we derive from plants – into an easily accessible global pool of

genetic resources that is freely available to potential users in the Treaty’s ratifying

nations for some uses.

Access and benefit sharing

• The Treaty facilitates access to the genetic materials of the 64 crops in the Multilateral

System for research, breeding and training for food and agriculture. Those who access

the materials must be from the Treaty’s ratifying nations and they must agree to use

the materials totally for research, breeding and training for food and agriculture. The

Treaty prevents the recipients of genetic resources from claiming intellectual property

rights over those resources in the form in which they received them, and ensures that

access to genetic resources already protected by international property rights is

consistent with international and national laws.

• Those who access genetic materials through the Multilateral System agree to share

any benefits from their use through four benefit-sharing mechanisms established by

the Treaty.

Page 86: SIMPLYFYING IAS EXAM PREPARATION - INSIGHTSIAS · Insta 75 Days Revision Plan for UPSC Civil Services Prelims – 2020 This document is the compilation of 100 questions that are part

INSTA 75 Days REVISION PLAN for Prelims 2020 - InstaTests

www.insightsonindia.com 84 Insights IAS

International Seed Treaty

• International Treaty of Plant Genetic Resources for Food and Agriculture (ITPGRFA), is

popularly known as the International Seed Treaty.

• It is a comprehensive international agreement, which aims at guaranteeing food

security through the conservation, exchange and sustainable use of the world’s plant

genetic resources for food and agriculture (PGRFA).

It also recognizes farmers’ rights, subject to national laws to:

• The protection of traditional knowledge relevant to plant genetic resources for food

and agriculture;

• The right to equitably participate in sharing benefits arising from the utilization of

plant genetic resources for food and agriculture.

• The right to participate in making decisions, at the national level, on matters related

to the conservation and sustainable use of plant genetic resources for food and

agriculture.

• The Treaty establishes the Multilateral System of Access and Benefit-sharing to

facilitate plant germplasm exchanges and benefit sharing through Standard Material

Transfer Agreement (SMTA).

• The treaty was negotiated by the Food and Agriculture Organization of the United

Nations (FAO) Commission on Genetic Resources for Food and Agriculture (CGRFA)

and since 2006 has its own Governing Body under the aegis of the FAO.

• The Governing Body is the highest organ of the Treaty and holds biennial meetings.

85. A new type of El Nino called El Nino Modoki appeared in the news. In this context,

consider the following statements:

1. Normal El Nino forms in the Central Pacific ocean whereas El Nino Modoki forms

in Eastern Pacific ocean.

2. Normal El Nino results in diminished hurricanes in the Atlantic Ocean but El Nino

Modoki results in a greater number of hurricanes with greater frequency.

Which of the statements given above is/are correct?

(a) 1 only

(b) 2only

(c) Both 1 and 2

(d) Neither I nor 2

Solution: B

Page 87: SIMPLYFYING IAS EXAM PREPARATION - INSIGHTSIAS · Insta 75 Days Revision Plan for UPSC Civil Services Prelims – 2020 This document is the compilation of 100 questions that are part

INSTA 75 Days REVISION PLAN for Prelims 2020 - InstaTests

www.insightsonindia.com 85 Insights IAS

• El Niño Modoki is a coupled ocean-atmosphere phenomenon in the tropical Pacific. It

is different from another coupled phenomenon in the tropical Pacific namely, El Niño.

• Conventional El Niño is characterized by strong anomalous warming in the eastern

equatorial Pacific. Whereas, El Niño Modoki is associated with strong anomalous

warming in the central tropical Pacific and cooling in the eastern and western tropical

Pacific

• Normal El Nino results in diminished hurricanes in the Atlantic Ocean but El Nino

Modoki results in a greater number of hurricanes with greater frequency. (UPSC

2010)

86. Consider the following statements regarding Mission 175

1. Mission 175 is a global platform to explore strategies for development and

deployment of renewables.

2. It is hosted by the Ministry of New and Renewable Energy (MNRE).

Which of the statements given above is/are correct?

(a) 1 only

(b) 2 only

(c) Both 1 and 2

(d) Neither 1 nor 2

Solution: C

Mission 175

• Mission 175 is a global platform to explore strategies for development and

deployment of renewables.

• It is hosted by the Ministry of New and Renewable Energy (MNRE), the event

showcases India’s renewable energy potential and the Government’s efforts to scale

up capacity to meet the national energy requirement in a socially, economically and

ecologically sustainable manner.

• In the year 2015, the Government of India announced a target for 175 GW cumulative

renewable power installed capacity by the year 2022.

• Ahead of COP 21, India submitted its Intended Nationally Determined Contribution

(INDC) to the UNFCCC, outlining the country’s post-2020 climate actions.

• India’s INDC builds on its goal of installing 175 Gigawatts (GW) of renewable power

capacity by 2022 by setting a new target to increase the country’s share of non-fossil

based installed electric capacity to 40 percent by 2030.

Page 88: SIMPLYFYING IAS EXAM PREPARATION - INSIGHTSIAS · Insta 75 Days Revision Plan for UPSC Civil Services Prelims – 2020 This document is the compilation of 100 questions that are part

INSTA 75 Days REVISION PLAN for Prelims 2020 - InstaTests

www.insightsonindia.com 86 Insights IAS

87. Consider the following statements regarding Bonn Convention

1. Conference of Parties (COP) 13 of the Convention on the Conservation of

Migratory Species of Wild Animals (CMS) was organized by India at Chilika lake.

2. The mascot of the event was Gibi, The Great Indian Bustard.

3. The Indian sub-continent is part of the major bird flyway network, i.e, the Central

Asian Flyway (CAF).

Which of the statement above is/are correct?

(a) 1 and 2 only

(b) 2 and 3 only

(c) 3 only

(d) 1, 2 and 3

Solution: B

Convention on the Conservation of Migratory Species of Wild Animals

• The Convention on the Conservation of Migratory Species of Wild Animals, also known

as the Convention on Migratory Species or the Bonn Convention.

• It is an international agreement that aims to conserve migratory species within their

migratory ranges

• Conference of Parties (COP) 13 of the Convention on the Conservation of Migratory

Species of Wild Animals (CMS) was organized from February 17 to 22 in Gandhinagar,

Gujarat.

• CMS is an environment treaty under the United Nations Environment Programme

(UNEP).

• There are 130 parties to the convention and India has been a member since 1983.

• India hosted the 13th COP of CMS at Gandhinagar, Gujarat.

• Theme – ―Migratory species connect the planet and we welcome them home

• Mascot – ―Gibi – The Great Indian Bustard

Highlights

• It is the first of a series of international nature-related meetings in 2020 which adopted

a new global biodiversity strategy for the next decade – Post-2020 Global Biodiversity

Framework

• In COP 13, the first ever report on the Status of Migratory Species was presented.

• It adopted the Gandhinagar Declaration which calls for migratory species and the

concept of ‗ecological connectivity‘ to be integrated and prioritized in the new

Framework.

Page 89: SIMPLYFYING IAS EXAM PREPARATION - INSIGHTSIAS · Insta 75 Days Revision Plan for UPSC Civil Services Prelims – 2020 This document is the compilation of 100 questions that are part

INSTA 75 Days REVISION PLAN for Prelims 2020 - InstaTests

www.insightsonindia.com 87 Insights IAS

• CMS Ambassadors Programme was relaunched for terrestrial, avian and aquatic

species.

• Seven Migratory Species Champions – Germany, India, Italy, Monaco, Norway, the

European Commission, and the Environment Agency were acknowledged for their

generous contributions to CMS initiatives.

• Ethiopia joined the CMS Memorandum of Understanding on the Conservation of

Migratory Birds of Prey in Africa and Eurasia (Raptors MOU).

• Ethiopia is a strategically important country for the conservation of migratory birds of

prey given its location across the East African flyway.

• This was the first CMS COP to be inaugurated by a host-country Head of

Government.

• India has been designated the President of the COP for the next three years (till

2023).

• India is home to several migratory species of wildlife, including the snow leopard,

Amur falcons, bar- headed geese, black-necked cranes, marine turtles, dugongs and

hump-backed whales.

• The government of India has been taking necessary actions to protect and conserve

migratory marine species.

• Seven species that include Dugong, Whale Shark, Marine Turtle (two species), have

been identified for preparation of Conservation and Recovery Action Plan.

India is temporary home to several migratory animals and birds. The important among these

include Amur Falcons, Bar headed Geese, Black necked cranes, Marine turtles, Dugongs,

Humpbacked Whales, etc. The Indian sub-continent is also part of the major bird flyway

network, i.e, the Central Asian Flyway (CAF) that covers areas between the Arctic and Indian

Oceans, and covers at least 279 populations of 182 migratory water bird species, including 29

globally threatened species. India has also launched the National Action Plan for conservation

of migratory species under the Central Asian Flyway.

88. Consider the following Statements:

1. The boundaries of a National Park is fixed by state government

2. A Biosphere Reserve is declared to conserve a few specific species of flora and

fauna.

3. In a Wildlife Sanctuary, limited biotic interference is permitted.

Which of the statements given above is/are correct?

(a) 1 only

(b) 2 and 3 only

(c) 1 and 3 only

(d) 1, 2 and 3

Page 90: SIMPLYFYING IAS EXAM PREPARATION - INSIGHTSIAS · Insta 75 Days Revision Plan for UPSC Civil Services Prelims – 2020 This document is the compilation of 100 questions that are part

INSTA 75 Days REVISION PLAN for Prelims 2020 - InstaTests

www.insightsonindia.com 88 Insights IAS

Solution: C

• The state government can fix and alter boundaries of the National Parks with prior

consultation and approval with National Board of Wildlife. There is no need to pass

an act for alternation of boundaries of National Parks. No human activities are

permitted in a National Park.

• The major objectives of Biosphere Reserves are: (i) To conserve diversity and integrity

of plants, animals and micro-organism; (ii) To promote research on ecological

conservation and other environmental aspects and; (iii) To provide facilities for

education, awareness and training.

• In a Wildlife Sanctuary, limited biotic interference is permitted. Operations such as

harvesting of timber, a collection of minor forest products and private ownership

rights are permitted so long as they do not interfere with the well-being of animals. In

other words, limited biotic interference is allowed. (UPSC 2010)

89. Consider the following statements regarding Tropical Forest Alliance 2020 (TFA 2020)

1. It is a multi-stakeholder partnership platform, initiated to support the

implementation of private-sector commitments to remove deforestation from

palm oil, beef, soy, and pulp and paper from their supply chains.

2. It is hosted by the World Economic Forum.

3. The Tropical Forest Alliance was founded in 2012 at Rio+20.

Which of the statements given above is/are correct?

(a) 1 and 2 only

(b) 2 and 3 only

(c) 1 and 3 only

(d) 1, 2 and 3

Solution: D

• The Tropical Forest Alliance (TFA) is a multi-stakeholder partnership platform,

initiated to support the implementation of private-sector commitments to remove

deforestation from palm oil, beef, soy, and pulp and paper from their supply chains.

Hosted by the World Economic Forum, it has grown its partner members throughout

the years and continues to bring on board those key actors committed to tackling

deforestation. TFA now works with governments, the private sector and civil society

actors, indigenous peoples, communities and international organizations to catalyse

Page 91: SIMPLYFYING IAS EXAM PREPARATION - INSIGHTSIAS · Insta 75 Days Revision Plan for UPSC Civil Services Prelims – 2020 This document is the compilation of 100 questions that are part

INSTA 75 Days REVISION PLAN for Prelims 2020 - InstaTests

www.insightsonindia.com 89 Insights IAS

high-impact partnerships to reduce commodity driven deforestation and build a forest

positive future.

TFA Global and at the Forest Frontier

• The strength of TFA as a platform is to bring together different communities, to

identify key forest frontier challenges and solutions. Through bringing experts from

around the world our community of purpose turns ideas into action, both at the global

and the regional level. It operates regional platforms in Latin America, West and

Central Africa, China and Southeast Asia. TFA’s work will also focus on mainstreaming

“Forest Positive” landscapes and amplify demand side engagement in major

economies such as the US, Europe and in China.

Our Donors

• We are grateful for the support of our donors who provide TFA with the necessary

resources to support our work. Our funders currently include the Governments of the

Netherlands, Norway, Germany, the United Kingdom, and the Gordon and Betty

Moore Foundation.

• The TFA catalyzes the power of collective action and responsibility by convening,

curating and communicating.

Our History

• The Tropical Forest Alliance was founded in 2012 at Rio+20 after the Consumer Goods

Forum (CGF) committed to zero net deforestation by 2020 for palm oil, soy, beef, and

paper and pulp supply chains in 2010. The CGF partnered with the US government to

create the public-private alliance with the mission of mobilizing all actors to

collaborate in reducing commodity-driven tropical deforestation.

Accelerating into the Decade of Delivery

• TFA fosters cross-sector collaboration based on a common and ever-deepening

understanding of the barriers and opportunities linked to deforestation-free supply

chains.

• Its greatest offering is a partnership of champions for deforestation-free global and

local economies, making the case for sustainable supply chains as an essential

pathway towards a better economy and achievement of the Global Goals.

90. Which of the following regions have been identified as bloom hotspots

1. North Eastern Arabian Sea

2. Coastal waters off Kerala

3. Gulf of Mannar

4. Coastal waters of Gopalpur

Select the correct answer using the code given below:

Page 92: SIMPLYFYING IAS EXAM PREPARATION - INSIGHTSIAS · Insta 75 Days Revision Plan for UPSC Civil Services Prelims – 2020 This document is the compilation of 100 questions that are part

INSTA 75 Days REVISION PLAN for Prelims 2020 - InstaTests

www.insightsonindia.com 90 Insights IAS

(a) 1, 2 and 3 only

(b) 2, 3 and 4 only

(c) 1, 3 and 4 only

(d) 1, 2, 3 and 4

Solution: D

Algal Bloom Information Service (ABIS)

• The increasing frequency of algal blooms is a major concern due to its ill effects on

fishery, marine life and water quality. INCOIS has developed a service for “Detection

and Monitoring of Bloom in the Indian Seas”.

• The target users are fishermen, marine fishery resource managers, researchers,

ecologists and environmentalists.

• The service also complements INCOIS’ marine fishing advisories i.e. Potential Fishing

Zone advisories.

• INCOIS-ABIS will provide near-real time information on spatio-temporal occurrence

and spread of phytoplankton blooms over the North Indian Ocean.

• Accordingly, relevant data retrieved from satellites i.e. Sea Surface Temperature ,

chlorophyll-a, Algal Bloom Index – chlorophyll, rolling chlorophyll anomaly, rolling

sea surface temperature anomaly, phytoplankton class/species, phytoplankton size

class and a composite image delineating bloom and non-bloom regions will be

disseminated daily through ABIS.

• In addition, four regions have been identified as bloom hotspots viz.

o North Eastern Arabian Sea

o coastal waters off Kerala

o Gulf of Mannar and

o Coastal waters of Gopalpur.

91. Consider the following statements regarding Foreigners Tribunals

1. The foreigners tribunals are quasi-judicial bodies,

2. The powers to constitute foreigners tribunals are vested only with the Centre.

Which of the statements given above is/are correct?

(a) 1 only

(b) 2 only

(c) Both 1 and 2

(d) Neither 1 nor 2

Page 93: SIMPLYFYING IAS EXAM PREPARATION - INSIGHTSIAS · Insta 75 Days Revision Plan for UPSC Civil Services Prelims – 2020 This document is the compilation of 100 questions that are part

INSTA 75 Days REVISION PLAN for Prelims 2020 - InstaTests

www.insightsonindia.com 91 Insights IAS

Solution: A

Foreigners Tribunals

• In 1964, the govt brought in the Foreigners (Tribunals) Order.

• Composition: Advocates not below the age of 35 years of age with at least 7 years of

practice (or) Retired Judicial Officers from the Assam Judicial Service (or) Retired IAS

of ACS Officers (not below the rank of Secretary/Addl. Secretary) having experience in

quasi-judicial works.

Who can setup these tribunals?

• The Ministry of Home Affairs (MHA) has amended the Foreigners (Tribunals) Order,

1964, and has empowered district magistrates in all States and Union Territories to

set up tribunals (quasi-judicial bodies) to decide whether a person staying illegally in

India is a foreigner or not.

• Earlier, the powers to constitute tribunals were vested only with the Centre.

• Typically, the tribunals there have seen two kinds of cases: those concerning persons

against whom a reference has been made by the border police and those whose

names in the electoral roll has a “D”, or “doubtful”, marked against them.

Who can approach?

• The amended order (Foreigners (Tribunal) Order, 2019) also empowers individuals to

approach the Tribunals. Earlier, only the State administration could move the Tribunal

against a suspect.

How a person is declared foreigner by these tribunals?

• Foreigners Tribunals, quasi-judicial authorities in Assam, have been deciding on

matters pertaining to citizenship in order to identify foreigners.

• The process begins by the border police or the Election Commission referring the case

of a suspected foreigner to the Foreigners Tribunal.

• The tribunal calls on the person to appear before it and prove that they are not a

foreigner, and then passes an order in favour or against them.

92. Consider the following statements regarding Nano Mission

1. The International Conference on Nano Science and Nano Technology (ICONSAT)

was organized under the aegis of Nano Mission.

2. Nano Mission Council chaired by Prime Minister.

3. It is as an umbrella capacity-building programme.

Which of the statements given above is/are correct?

(a) 1 and 2 only

Page 94: SIMPLYFYING IAS EXAM PREPARATION - INSIGHTSIAS · Insta 75 Days Revision Plan for UPSC Civil Services Prelims – 2020 This document is the compilation of 100 questions that are part

INSTA 75 Days REVISION PLAN for Prelims 2020 - InstaTests

www.insightsonindia.com 92 Insights IAS

(b) 3 only

(c) 1 and 3 only

(d) 1, 2 and 3

Solution: C

• The International Conference on Nano Science and Nano Technology (ICONSAT) under

the aegis of Nano Mission, Department of Science and Technology (DST) is being held

at Kolkata focusing on the recent advances in this frontier research field.

Mission on Nano Science and Technology (Nano Mission):

• Launched in 2007.

• It is as an “umbrella capacity-building programme”.

• The Mission’s programmes will target all scientists, institutions and industry in the

country.

• It will also strengthen activities in nano science and technology by promoting basic

research, human resource development, research infrastructure development,

international collaborations, among others.

• It will be anchored in the Department of Science and Technology and steered by a

Nano Mission Council chaired by an eminent scientist.

Outcomes and significance of the mission:

• As a result of the efforts led by the Nano Mission, today, India is amongst the top five

nations in the world in terms of scientific publications in nano science and technology

(moving from 4th to the 3rd position).

• The Nano Mission itself has resulted in about 5000 research papers and about 900

Ph.Ds and also some useful products like nano hydrogel based eye drops, pesticide

removal technology for drinking water, water filters for arsenic and fluoride removal,

nanosilver based antimicrobial textile coating, etc.

• The Nano Mission has thus helped establish a good eco-system in the country to

pursue front-ranking basic research and also to seed and nurture application-oriented

R&D, focused on useful technologies and products.

93. Consider the following statements regarding Gadgil and Kasturi Rangan committees

1. The Kasturi Rangan report seeks to bring 67% of the Western Ghats under the

Ecologically Sensitive Area (ESA) zones.

2. The Gadgil report seeks to bring 37% of the Western Ghats under the Ecologically

Sensitive Area (ESA) zones.

Which of the statements given above is/are correct?

Page 95: SIMPLYFYING IAS EXAM PREPARATION - INSIGHTSIAS · Insta 75 Days Revision Plan for UPSC Civil Services Prelims – 2020 This document is the compilation of 100 questions that are part

INSTA 75 Days REVISION PLAN for Prelims 2020 - InstaTests

www.insightsonindia.com 93 Insights IAS

(a) 1 only

(b) 2 only

(c) Both 1 and 2

(d) Neither 1 nor 2

Solution: D

• A public interest litigation petition has been filed in the Madras High Court seeking a

direction to the Centre and State government to constitute a permanent body for

taking serious steps to safeguard the flora, fauna and other natural resources in the

Eastern and Western Ghat areas in Tamil Nadu.

• The petition is on the basis of the recommendations made by the Madhav Gadgil and

Kasturi Rangan committees.

What’s the issue? Why there is a need for protection?

• Petitioner contended that the natural resources abundantly available in this area are

being properly utilized by other regions, except Tamil Nadu. They are being misutilised

and mismanaged not only by the administrators but also by the public at large.

• Besides, large-scale plantations of coffee, tea and orchards have been raised in the

hills of Western Ghats. Aromatic and valuable trees like sandal are removed illegally.

Despite the Wildlife Protection Act, hunting takes place in some pockets. The forests

are getting degraded because of illicit collection of firewood, illicit grazing and illicit

felling of trees.

What did the Gadgil Committee say?

• It defined the boundaries of the Western Ghats for the purposes of ecological

management.

• It proposed that this entire area be designated as ecologically sensitive area (ESA).

• Within this area, smaller regions were to be identified as ecologically sensitive zones

(ESZ) I, II or III based on their existing condition and nature of threat.

• It proposed to divide the area into about 2,200 grids, of which 75 per cent would fall

under ESZ I or II or under already existing protected areas such as wildlife sanctuaries

or natural parks.

• The committee proposed a Western Ghats Ecology Authority to regulate these

activities in the area.

Why was Kasturirangan Committee setup?

• None of the six concerned states agreed with the recommendations of the Gadgil

Committee, which submitted its report in August 2011.

Page 96: SIMPLYFYING IAS EXAM PREPARATION - INSIGHTSIAS · Insta 75 Days Revision Plan for UPSC Civil Services Prelims – 2020 This document is the compilation of 100 questions that are part

INSTA 75 Days REVISION PLAN for Prelims 2020 - InstaTests

www.insightsonindia.com 94 Insights IAS

• In August 2012, then Environment Minister constituted a High-Level Working Group

on Western Ghats under Kasturirangan to “examine” the Gadgil Committee report in

a “holistic and multidisciplinary fashion in the light of responses received” from states,

central ministries and others.

• The Kasturirangan report seeks to bring just 37% of the Western Ghats under the

Ecologically Sensitive Area (ESA) zones — down from the 64% suggested by the Gadgil

report.

Recommendations of Kasturirangan Committee:

• A ban on mining, quarrying and sand mining.

• No new thermal power projects, but hydro power projects allowed with restrictions.

• A ban on new polluting industries.

• Building and construction projects up to 20,000 sq m was to be allowed but townships

were to be banned.

• Forest diversion could be allowed with extra safeguards.

94. Consider the following statements regarding India Urban Data Exchange (IUDX)

1. It has been launched under smart cities mission

2. It is being implemented by Ministry of Statistics and Programme Implementation

3. It aims to enabling cities to harness full potential of the enormous data being

generated in our smart cities.

Which of the statements given above is/are correct?

(a) 1 only

(b) 2 and 3 only

(c) 1 and 3 only

(d) 1, 2 and 3

Solution: C

India Urban Data Exchange (IUDX)

• IUDX is a research project under smart cities mission being implemented by Union

Ministry of Housing and Urban Affairs (MoHUA) in collaboration with Indian Institute

of Science (IISc), Bengaluru.

• It is an open source software platform aimed at enabling cities to harness full

potential of the enormous data being generated in our smart cities

• The Ministry of Housing and Urban Affairs (MoHUA) launched an ambitious program

for the development of 100 Smart Cities aimed at catalyzing investments in holistic

Page 97: SIMPLYFYING IAS EXAM PREPARATION - INSIGHTSIAS · Insta 75 Days Revision Plan for UPSC Civil Services Prelims – 2020 This document is the compilation of 100 questions that are part

INSTA 75 Days REVISION PLAN for Prelims 2020 - InstaTests

www.insightsonindia.com 95 Insights IAS

infrastructure and services, development of integrated approaches to problem

solving, promotion of co-creation and bottom-up innovation using technology, and

creation of sustainable outcomes by espousing the circular economy and participatory

processes built around communities.

• The total investment expected in these cities over a period of 5 years from the date of

their selection is expected to be around US$ 30 billion.

• Smart Cities, among their many objectives, are evolving as connected, livable, energy

efficient, adaptive and resilient cities through deployment of smart solutions.

95. Consider the following pairs of the Ramsar wetlands and the the states they belong to:

Ramsar wetlands State 1. Bhoj Wetlands : Madhya Pradesh 2. Chandertal Wetland : Himachal Pradesh 3. Deepor Beel : Assam 4. Surinsar-Mansar Lakes : Jammu and Kashmir

Which of the pairs given above is/are correctly matched?

(a) 1, 2 and 3 only

(b) 2, 3 and 4 only

Page 98: SIMPLYFYING IAS EXAM PREPARATION - INSIGHTSIAS · Insta 75 Days Revision Plan for UPSC Civil Services Prelims – 2020 This document is the compilation of 100 questions that are part

INSTA 75 Days REVISION PLAN for Prelims 2020 - InstaTests

www.insightsonindia.com 96 Insights IAS

(c) 1, 3 and 4 only

(d) 1, 2, 3 and 4

Solution: D

RAMSAR WETLANDS SITES (Updated on February, 2019)

Sl. No. Name of Site State Location

1 Asthamudi Wetland Kerala

2 Bhitarkanika Mangroves Orissa

3 Bhoj Wetlands Madhya Pradesh

4 Chandertal Wetland Himachal Pradesh

5 Chilka Lake Orissa

6 DeeporBeel Assam

7 East Calcutta Wetlands West Bengal

8 Harike Lake Punjab

9 Hokera Wetland Jammu and Kashmir

10 Kanjli Lake Punjab

11 Keoladeo Ghana NP Rajasthan

12 Kolleru Lake Andhra Pradesh

13 Loktak Lake Manipur

14 Nalsarovar Bird Sanctuary Gujarat

15 Point Calimere Tamil Nadu

16 Pong Dam Lake Himachal Pradesh

17 Renuka Wetland Himachal Pradesh

18 Ropar Lake Punjab

19 Rudrasagar Lake Tripura

20 Sambhar Lake Rajasthan

21 Sasthamkotta Lake Kerala

22 Sunderbans Wetland West Bengal

Page 99: SIMPLYFYING IAS EXAM PREPARATION - INSIGHTSIAS · Insta 75 Days Revision Plan for UPSC Civil Services Prelims – 2020 This document is the compilation of 100 questions that are part

INSTA 75 Days REVISION PLAN for Prelims 2020 - InstaTests

www.insightsonindia.com 97 Insights IAS

23 Surinsar-Mansar Lakes Jammu and Kashmir

24 Tsomoriri Lake Jammu and Kashmir

25 Upper Ganga River (Brijghat to Narora Stretch) Uttar Pradesh

26 VembanadKol Wetland Kerala

27 Wular Lake Jammu & Kashmir

96. Houthis insurgents often seen in the news, are mainly found in

(a) Kenya

(b) Saudi Arabia

(c) Afghanistan

(d) Yemen

Solution: D

• The Houthi movement, officially called Ansar Allah and colloquially simply Houthis, is

an Islamic political and armed movement that emerged from Sa’dah in northern

Yemen in the 1990s. The movement was called Houthis because its founder is from

the Houthi tribe.

97. Consider the following statements regarding Census of Marine Life

1. It is a 10-year international effort undertaken in to assess the diversity (how many

different kinds), distribution (where they live), and abundance (how many) of

marine life.

2. The census is supported by funding from Royal Institution in London.

3. The world’s first comprehensive Census of Marine Life was released in 2020.

Which of the statements given above is/are correct?

(a) 1 only

(b) 2 and 3 only

(c) 1 and 3 only

(d) 1, 2 and 3

Solution: A

Page 100: SIMPLYFYING IAS EXAM PREPARATION - INSIGHTSIAS · Insta 75 Days Revision Plan for UPSC Civil Services Prelims – 2020 This document is the compilation of 100 questions that are part

INSTA 75 Days REVISION PLAN for Prelims 2020 - InstaTests

www.insightsonindia.com 98 Insights IAS

A DECADE OF DISCOVERY

• 2,700 scientists

• 80+ nations

• 540 expeditions

• US$ 650 million

• 2,600+ scientific publications

• 6,000+ potential new species

• 30 million distribution records and counting

These numbers only begin to describe the scope of the Census of Marine Life, a 10-year

international effort undertaken in to assess the diversity (how many different kinds),

distribution (where they live), and abundance (how many) of marine life—a task never before

attempted on this scale. The Census stimulated the discipline of marine science by tackling

these issues globally, and engaging some 2,700 scientists from around the globe, who

participated in 540 expeditions and countless hours of land-based research. The scientific

results were reported on October 4, 2010 at the Royal Institution in London.

The world’s first comprehensive Census of Marine Life — past, present, and future — was

released in 2010 in London. Initially supported by funding from the Alfred P. Sloan

Foundation, the project was successful in generating many times that initial investment in

additional support and substantially increased the baselines of knowledge in often

underexplored ocean realms, as well as engaging over 2,700 different researchers for the first

time in a global collaborative community united in a common goal, and has been described

as “one of the largest scientific collaborations ever conducted”.

98. Consider the following statements regarding Sabka Vishwas Scheme

1. It has been launched by Ministry of Social Justice and Empowerment

2. It aims to provide basic amenities to vulnerable sections.

Which of the statements given above is/are correct?

(a) 1 only

(b) 2 only

(c) Both 1 and 2

(d) Neither 1 nor 2

Solution: D

Page 101: SIMPLYFYING IAS EXAM PREPARATION - INSIGHTSIAS · Insta 75 Days Revision Plan for UPSC Civil Services Prelims – 2020 This document is the compilation of 100 questions that are part

INSTA 75 Days REVISION PLAN for Prelims 2020 - InstaTests

www.insightsonindia.com 99 Insights IAS

Sabka Vishwas Scheme

• It is a Legacy Dispute Resolution Scheme.

• The scheme targets those taxpayers who want to close their pending disputes related

to Service Tax and Excise Tax (now subsumed under Goods and Services Tax). Hence,

the term ‘legacy’.

• This scheme offers amnesty (official pardon) to those who wish to disclose any

previously undisclosed tax liability without any penalty or prosecution.

• Finance minister Nirmala Sitharaman had unveiled the scheme in the budget for 2019-

20, with an aim to assist taxpayers in clearing the baggage of disputes under legacy

taxes (service tax and central excise), which are subsumed in the goods and services

tax (GST).

99. Which of the following is/are Natural world heritage sites of India?

1. Great Nicobar National Park

2. Kaziranga National Park

3. Nanda Devi and Valley of Flowers National Parks

4. Sundarbans National Park

Select the correct answer using the code given below:

(a) 1, 2 and 3 only

(b) 2, 3 and 4 only

(c) 1, 3 and 4 only

(d) 1, 2, 3 and 4

Solution: B

• A UNESCO World Heritage Site is a place that is listed by the United Nations

Educational, Scientific and Cultural Organization as of special cultural or physical

significance.

Natural (7)

• Great Himalayan National Park Conservation Area (2014)

• Kaziranga National Park (1985)

• Keoladeo National Park (1985)

• Manas Wildlife Sanctuary (1985)

• Nanda Devi and Valley of Flowers National Parks (1988,2005)

• Sundarbans National Park (1987)

• Western Ghats (2012)

Page 102: SIMPLYFYING IAS EXAM PREPARATION - INSIGHTSIAS · Insta 75 Days Revision Plan for UPSC Civil Services Prelims – 2020 This document is the compilation of 100 questions that are part

INSTA 75 Days REVISION PLAN for Prelims 2020 - InstaTests

www.insightsonindia.com 100 Insights IAS

Mixed (1)

• Khangchendzonga National Park (2016)

http://www.wiienvis.nic.in/Database/whs_pas_8227.aspx

100. Consider the following statements regarding Pradhan Mantri Laghu Vyapari Maan-dhan

Yojana:

1. It is a voluntary and contribution based central sector scheme.

2. All small shopkeepers, self-employed persons and retail traders aged between 18-

40 years and with Goods and Service Tax turnover below Rs.1.5 crore can enroll

for this scheme.

3. Under this scheme, all beneficiaries eligible for a monthly pension of Rs 3000 after

the age of 60.

Which of the statements given above is/are correct?

(a) 1 and 2 only

(b) 2 and 3 only

(c) 1 and 3 only

(d) 1, 2 and 3

Solution: D

Pradhan Mantri Laghu Vyapari Maan-dhan Yojana:

• It is a voluntary and contribution based central sector scheme.

• The government launched the scheme, entailing monthly minimum assured pension

of ₹3,000 for the entry age group of 18-40 years after attaining the age of 60 years,

with effect from July 22, 2019.

• Under the scheme, the government makes matching contribution in the subscribers’

account.

• The scheme is based on self-declaration as no documents are required except bank

account and Aadhaar Card.

Eligibility:

• All small shopkeepers, self-employed persons and retail traders aged between 18-40

years and with Goods and Service Tax (GST) turnover below Rs.1.5 crore can enroll for

pension scheme.

Page 103: SIMPLYFYING IAS EXAM PREPARATION - INSIGHTSIAS · Insta 75 Days Revision Plan for UPSC Civil Services Prelims – 2020 This document is the compilation of 100 questions that are part

INSTA 75 Days REVISION PLAN for Prelims 2020 - InstaTests

www.insightsonindia.com 101 Insights IAS

• To be eligible, the applicants should not be covered under the National Pension

Scheme, Employees’ State Insurance Scheme and the Employees’ Provident Fund or

be an Income Tax assessee.

Page 104: SIMPLYFYING IAS EXAM PREPARATION - INSIGHTSIAS · Insta 75 Days Revision Plan for UPSC Civil Services Prelims – 2020 This document is the compilation of 100 questions that are part

www.insightsonindia.com INSIGHTS IAS

INSTA STORIES

Page 105: SIMPLYFYING IAS EXAM PREPARATION - INSIGHTSIAS · Insta 75 Days Revision Plan for UPSC Civil Services Prelims – 2020 This document is the compilation of 100 questions that are part

www.insightsonindia.com INSIGHTS IAS